Sie sind auf Seite 1von 38

1

Task
PSYCHIATRY a. How will you manage Margaret (lack of sleep, good
appetite, no early morning awakening, mood is okay,
Taking a psychiatric history: no psychosis)
- ENSURE confidentiality
- IF uncooperative: I understand that you are going History
through a bad time but please help me if you want me - Consent
to help you. - I understand that you have tiredness and feeling
- HPI: when? How? Worsening or getting better? What fatigue, can you tell me more about it? Do you have
is the effect on your life because of the symptom? any weather preference? Do you have palpitations?
What is the effect on your sleep? Light-headedness or dizziness? SOB? Chest pain?
- Mood: How is your mood? Have you noticed any Weight loss? Night sweats? Headaches? Any lumps
change in your sleep (in depression: early morning and bumps on the body? Any cough? Any tummy pain
awakening; anxiety: difficulty initiating sleep)? or change in waterworks or bowel motions? I
Change in appetite? Change in weight? Did you understand you smoke and drink alcohol, have you
experience any weight loss or weight gain (typical: tried using illicit drug use?
vegetative sx go down; atypical: vegetative sx go up); - Hows your mood? Has there ever been a time when
What is your energy level? Do you think life is worth your mood was very high? Any problems with sleep?
living? Have you thought of harming yourself or Weight? Appetite? Do you still find things
anybody else (What, when, how)? Are there times pleasurable? Do you think life is worth living? Have
when your mood is high? you thought about harming yourself or others? I
o anhedonia: loss of interest in activity which understand you were prescribed antidepressants
used to be pleasurable. before, do you know why it was given?
o Psychomotor retardation: more common in - Psychotic symptoms: Do you feel/see/hear things that
depression than agitation other do not? Do you have any strange experiences?
- Psychosis: Im sorry if I have to ask questions which - Hows your general health?
may sound silly but I will need your cooperation. Do
you see/hear/feel things which others do not? Did you Management
have any strange experiences? Do you think - Consider quitting one of the jobs
somebody is putting ideas in your head (thought - Consider moving out of the house
insertion)? Do you think your ideas are being - DO NOT prescribe antidepressants
broadcasted everywhere/do you think people are after - Lifestyle modification
your ideas (thought broadcasting)? Do you think that o Healthy diet
people, TV, radio, newspaper is talking about you? Do o Regular exercise
you hear voices telling you to harm o Address alcohol and smoking
yourself/somebody? - Meditation and yoga
- Insight: Do you think something is wrong with you? Do - Refer for stress management
you think you need help?
- Judgment: What will you do if there is a fire in this Acquired Brain Injury and behavioral changes
room? What will you do if you find an envelope with a
name, ticket, everything? Case: You are working in GP practice your next patient is a
- Cognition: do you know who you are? Where you are health worker who looks after a house accommodation disable
and the time? people. He is here to talk to you about James who is one of the
residents of the house. James is 37-years-old and living in this
ORGANIC DISORDERS AND DEMENTIAS house for a long time. He has a downs syndrome. He had a
head injury which required surgery when he was young. And
Lifestyle Stress then his family put him in this house as he needed a lot of
support. He is also having epilepsy which is well controlled with
Case: Margaret aged 35 years presents to your surgery on a the medication. The health worker is here to talk to you about
busy afternoon. She tells you she had about 6-8 weeks of Jamess recent change in behavior.
ongoing fatigue and tiredness. She denies any specific
symptoms but describes just fatigue, weariness, and feels as Task
cannot get out of her own way. Margaret is not your regular a. Talk to the health worker, Tim
patient but attended surgery on few occasions and you know b. Management
she had changed a couple of jobs but now tells you she is
working at 3 different places everyday starting from 8-7pm. She History: He was shouting at other residents and slams the door.
was in a relationship but broke up recently and at present is He hardly talks to anybody. Family talk to him but didnt come.
living with her extended family in suburban area of the city. He respond but doesnt want to talk to anyone. No fever, Used
Margaret is smoker for the last 15 years and on average to work. From last 4 weeks he cannot get up in the morning,
smokes about 15-20 cigarettes per day and drinks 3-4 standard become very abusive. Nothing happen at work.
drinks of red wine every day. She takes ASA for her occasional
tension headaches and takes no other OTC or prescribed Features:
medications. she had not other significant PMHx. Margaret - Acquired Brain Injury: Any type of the brain damage
describes some stress at work and finds hard to cope with the that occurs after birth.
manager at one of the jobs but still she is carrying on. She - The brain injury happens in two ways:
requests you to prescribe her some medications for her stress o Sudden onset: trauma, infection, lack of
and mentions that few years ago one of the GPs of same oxygen to brain. (Near drowning), Stroke.
practice described her antidepressants medications which she o Insidious onset: prolong alchol and
used for some time and thinks that maybe she needs those substances abuse, tumor or degenerative
meds again. diseases.
o How does it affect the person? Long term
effect of ABI are difficult to predict. Its
different in different people, the patient can
2

present with behavior and personality - PMHx: any history of heart condition? Hypertension?
changes. Thinking and learning abilities. SADMA? Whom do you live with? Do you have
Increase fatigue. enough support?

History Physical Examination


- Can you tell me more about what happened? He is - General appearance
violent, verbally abusive to staff, socially isolated, he - Vital signs
doesnt want to talk to anybody. They used to go to - Chest and heart examination
work, but last 4 weeks he doesnt want to go to work.
- Hows his mood? Anhedonia? Sleep? Weight? Diagnosis and Management
Appetite? try to self harm himself or somebody else? - I would like to admit the patient in the hospital now.
No he doesnt. For the chest pain, we will do ECG and cardiac
- Psychotic symptoms:Talks bizarre? Delusions? enzymes and take blood for baseline investigations. I
See/hear things others do not? Family support? SAD would call in the medical registrar for further
- I understand he had brain injury years ago, do you assessment to rule out any organic cause.
think something has happened recently (No)? How - Secondly, he may have relapse of your schizophrenia
about his epilepsy is it under control (Yes)? because he is not taking his medications and he has
nobody to take care of him at home. Based on these,
Management: well arrange for transfer to tertiary hospital for
- Alright Tim, Id like to see James. Id also organize psychiatric assessment and management.
psychiatrist. Im concerned that the personality
changes might be because of the acquired brain injury Dementia of Alzheimer Type
when he was young/in the past. We need to organize
some investigations. For example CT head, urine drug Case: A 35-year-old lady comes to your GP clinic asking about
screen, other blood tests. her father who has recently been diagnosed with Alzheimer
- Can you give some medications for his behavior now? disease. She is very concerned about her father and has many
No, not at this stage, psychiatrist needs to see him questions from you. The daughter has the fathers permission to
first before give him medications. Id like to involve the inquire about his condition.
family. If he has self harm we need to admit him. Task
- Explain about Alzheimer disease
Psychiatric Patients Get Sick Too - Answer her question

Case: A 40-years-old man comes to ED in a district hospital Counseling


where you work as HMO. He had chronic schizophrenia for the - What is Alzheimer disease? It is a type of dementia
last 20 years and now complains of pain in the chest. The where there is wasting of brain cells which in turn
tertiary care hospital where psychiatrist is available is 200km affect the function of the brain.
away. - The early features of this disease are:
o short-term memory loss (esp recent memory
Task where the person cannot remember what
a. History (chest pain since last 2 days, all over the chest has happened a few hours ago or even
4-5/10 in severity; not taking medications for the last 1 moments earlier but may clearly remember
year; I can see my girlfriend constricting my chest and the events of the past)
back; my dentist has inserted a magnet into my tooth o progressive decline in cognitive/mental
to spy on what Im doing; no suicidal ideations; lives functioning
alone and nobody to take care of him) - Unfortunately, it can worsen slowly with time and can
b. Physical examination (normal) lead to behavior changes and severe memory loss
c. Discuss diagnosis and management where the person might even forget the names of
family members. It may also lead to self-neglect that
History can further proceed to accidents at home or outside
- Is my patient hemodynamically stable? and poor nutrition.
- Confidentiality - Is the diagnosis 100% sure? It is a diagnosis of
- May I know a bit more about the chest pain? Since exclusion. It is only after excluding the correctable
when? Where exactly how severe is the pain? Do you causes that we can diagnose Alzheimer disease. We
need any pain killer? What is the type of pain? Does it might see some changes on the CT scan of the brain.
go anywhere else? Is it for the first time? Did the pain However, the definitive diagnosis can only be known
come with nausea/ vomiting, sweating, anxiety? Does after the person passes away and examining the
it come with any activity or walking? Do you feel any tissue of the brain under the microscope is done.
SOB, any racing of heart? Any pain associated with - Can it be depression? Depression by itself can
chest movement or breathing? Any recent URTI or produce dementia and it is called pseudodementia. It
fever? Any injury or trauma to your chest? Any heart is important to exclude it before diagnosing dementia.
burn or tummy pain? On the other hand, people with Alzheimer disease can
- I understand you had schizophrenia for the last 20 have depression, especially in the early stages of the
years, are you taking your medications regularly? disease because they can recognize their disability.
When did you last see the specialist? What - Do not worry so much. I will be visiting your father and
medications were you on? assess his condition and he will also be assessed by a
- Why did you stop your medications? Any side effects? psychiatrist, and if required, will be given medications
I would like to ask you some routine questions, do you for depression and he will also be followed-up on a
see/hear/feel things that other do not? How is your regular basis
mood? Any change is your weight, appetite, or sleep? - He will be assessed by age-care assessment team.
Do you think life is worth living? Have you thought of This team is composed of geriatrician, occupational
harming yourself or others? therapist, physiotherapist, social worker, myself as a
GP, psychologist and psychiatrist. The role of this
3

team is to assess your fathers condition, level of Due to poor Recall and Registration, patient might have
dependency, and eligibility for services that can be Alcohol-Induced Brain Injury
offered to him. Geriatrician might prescribe some
medications that can delay the progress but will not Wernicke-Korsakoff Syndrome
treat the illness. Occupational therapist can assess - Vision : Ophthalmoplegia
the home situation and his needs (eg fix lights, put - Ataxia
railings, remove loose carpets, etc) to keep your father - Memory Impairment (Amnesia)
safe. The social worker can arrange meals on wheels
if required, help him in washing the clothes and MMSE
cooking food, and can organize social support. - I will do MMSE which is a screening test to assess
Physiotherapist will assess his ability to walk and your cognitive or mental functioning. It is a simple test
might provide him with walking aids. Psychiatrist will that includes questions that assess you in a number of
assess his mental state and prescribe some areas and has thirty points in total. It will take
medications. approximately five minutes and I will guide you
- Is it better to put him in a nursing home? The aged- through it once we start. If you have any problem or
care assessment team will decide on it after questions, please dont hesitate to stop me. Can we
assessment and the options available are: do the MMSE?
o To stay home if he can cope (preferred
option due to familiar home environment). - Doing the MMSE.....(+) problem on registration and
He will be assessed regularly by team. recall.
o Living in the nursing home where a nurse - It can be Alcohol-induced Brain injury because of
will take care of him and I will also visit him chronic alcohol abuse. It can also be Wernicke-
regularly Korsakoff Syndrome.
o If at any time you want to take him back - What are you going to do with this patient? (by
home, you can do that and you can have Examiner)
access to respite care. It is a type of care - I will do complete medical evaluation and refer to
given by trained people on a temporary psychiatrist for further neuropsychological testing to
basis that help you take a break and have exclude more diffused impairment like dementia.
some rest. - Critical Error: Failure to identify short-term memory
- His vision and hearing will also be checked and his deficits; and a response that patient is delirious or
license may be suspended. It is very important for his demented.
safety and wellbeing.
- Will I get Alzheimer disease? It cannot be said at this CASE 109 Teaching Folstein MMSE
moment although there is a rare type of Alzheimer that
runs in families and occurs at an earlier age. But Case 1: You are a resident in Psychiatry Department. A 28-
because there is no family history, it may not be year-old was admitted with severe depression who is currently
possible. However, the specialist can explain more taking SSRIs. She is a secondary school teacher. A final year
about it. medical student did an MMSE and wants to discuss the results
which he shows on a small piece of paper.
Performing MMSE (Bookcase Condition 146)
Case 2: You are a resident in a large teaching hospital and you
Case: A 50-year-old barman comes to the GP clinic. He has a are asked by a final year medical student to teach you MMSE
history of consumption of up to 10 standard alcoholic drinks over and wants to discuss the results.
the last few weeks. His wife told you that he is quite forgetful
and unreliable for some months. You have completed the Orientation 5+5/10
history and now proceeding to test his cognitive function. Registration 0/3
Attention and Concentration 1/5
Task: Recall 0/3
a. Do MMSE Language 2+1+3+1 = 7/9
b. Explain what you are doing and why Did not write the sentence and copy the diagram : gave up
c. Summarize to the examiner the normal and abnormal Total 18/30
MMSE findings
d. Interpret the results to the examiner including what Tasks:
conditions these results signify a. Explain the results
b. Answer the questions that the student asks you
MMSE RESULTS of the patient: problem on registration and
recall therefore patient has short -term memory)
- ORIENTATION 5+5/10 MMSE (ORARLC) (total of 30 points)
- REGISTRATION 1/3 (3 tries) - Orientation (total of 10)
- ATTENTION AND CONCENTRATION (5/5) o 5 points: Year, Season, Date, Day, Month,
- RECALL 0/3 o 5 points: State City Suburb Hospital Floor,
- LANGUAGE Place
- Name two objects (2) - Registration (Immediate Memory; Total of 3)
- No, If's and or but (1) o Say three things and ask patient to repeat
- ask patient to close eyes (1) o Patient recalls three things
- write a sentence (1) o Patient can go for 6 tries
- hold paper with right hand, fold into half, put on lap (3) - Attention and Concentration (total of 5)
- CONSTRUCTION o WORLD Spell forward and Backward
- draw diagram (1) (easier to do and less time-consuming) or
o Subtract 7 starting from 100:
Patient is most likely not delirious because of intact orientation. - Recall (Short-term memory; Total of 3)
(Critical Error: if you say patient is delirious) o Reproduce three things that I have told you
Delirium is disorientation.... a while ago
4

- Language As an example, better educated people may score


o Name two objects (2) well on the test despite having significant cognitive
o No, If's and or but (1) impairment. Discuss MMSE scores...normal 25 to 30.
o Ask pt to close her eyes (1) - Does the result show that the patient has dementia?
o Can you write a sentence for me? (1) No dementia is a diagnosis of exclusion and requires
o Hold paper in right, fold it into half, put it on complete assessment with further neuropsychological
your lap (3) testing by the specialist before diagnosing it. In this
- Construction case, the patient was admitted with depression which
o Draw Diagram: Two overlapping hexagons sometimes can be severe enough to cause pseudo-
which are side by side (1) checks dementia. The score of 18/30 is most likely because of
neospatial orientation it. As you can see, the patients with depression gave
o Depression may have pseudo-dementia up even before finishing the examination; however the
patients with dementia will try hard to get a good score
and complete the examination.
MMSE - Can we repeat the test tomorrow? We can do it as it is
Formula : ORARLC brief and easily conducted test but it is better to wait
Dementia: earliest symptom is lost of short-term memory until her mood symptoms get resolved. It can take
three to 4 weeks for the drugs to work and then we
Registration and Recall is hallmark of dementia can repeat the test and the score should improve with
Delirium: has poor orientation, something acute and patient is the improvement of her depressive symptoms.
confused; auditory hallucinations.... - Can we adjust the score because she gave up on the
latter part of the test? No, we don't change the scores
Patient came in with 18/30 and had problems with registration, - What we do if she gets low score when her mood
recall and language symptoms are better? We will investigate her with full
dementia screening and further neuropsychological
Is this depression or dementia?? testing by a specialist.
MMSE is a screening tool not a diagnostic test for Delirium, - Can it be delirium? No the patient's orientation is
dementia or head injury. It is a bedside test. It is affected by normal and delirium has a rapid onset with a reduced
many factors like Education, ethnicity, speech, age, physical consciousness and a fluctuating course over the 24-
disabilities like hearing. Dementia is a Diagnosis of Exclusion. hour period. Delirium has a cause. They can have
Depression is pseudo-dementia hallucination but it is acute.
- Normal: 25 to 30.
- Mild-moderate impairment: 18-24 Frontal Lobe Dementia
- Severe: less than 18
Depression Case: A 50-year-old man is in your GP clinic. His wife visited
- is acute and past history. you already and told you that he changes the lane while driving
- Patient is agitated and does not want to cooperate without obvious reasons. He also has behavioral problems
because they are aware that something is wrong with recently. He is in your clinic because his wife insisted him for a
them. This patient gave up because she had insight. check up
- Recent and Remote affected MMSE is already done and no need to repeat it. In MMSE,
- They talk about their deficits RECALL is 0/3, Language When patient is asked to hold paper
- Has treatment and fold and put on his lap: 0/3
- Repeat MMSE the following day? No, results will be
the same. Improvement will be seen in 4 to six weeks Task:
- Is it dementia? Is it delirium? not delirium and not a. Take a further history
dementia b. Do at least 1 test to assess his cognitive function.
- Is it depression? YEs, Depression is pseudo- c. Discuss your provisional diagnosis with the examiner
dementia. and the reason for it.

Dementia (a diagnosis of exclusion; exclude causes by e.g. (At the EXAM, Two papers outside: 1st is the STEM, 2nd one is
Neuropsychological testing MMSE results)
- is slow, insidious and progressive
- No past history History
- Lack insight and confabulate. - How can I address you? (Why do you want to know
- Loss starts from recent then remote. my address?)
- Hide the deficits - Your wife is concerned about you. May I know why? (I
- No treatment have no problem)
- Did you notice any changes in behavior at home?
Counseling (No.) Any changes in your mood? Irritability? Any
- Do you have of any particular concern before I discuss problem with memory? Any problem with driving? Any
to you the result? problem in performing daily activities? Patient asks to
- MMSE is a bedside cognitive function screening test. repeat some questions.
Its purpose is not to make a diagnosis but to indicate -
the presence of cognitive impairment due to delirium, - Have you had head injury? How is your mood? History
dementia, or head injury. The advantage of this test is of stroke, heart attack? Taking any medications?
that it only takes 5 minutes which is therefore practical Smoking? alcohol? Drugs?
to use repeatedly and routinely. It can be helpful to
monitor the progress or fluctuation in these disorders History: (summary)
that may benefit from intervention. The disadvantage - Having short term memory problems and forgetting
is that it can be affected by age, years of education, many question. Patient had no insight. He was getting
socio-economic status, the background of the irritated and agitated. His understanding of simple
patient/ethnicity and physical problems like hearing. language was impaired?
5

EXAMINATION: Mnemonic: DEMENTIAS - Things to rule out before Dementia


can be diagnosed
DEMENTIA SCREENING TEST D - 3D's Dementia
- Clock face Drawing test (ask patient to draw a clock at Depression
ten past eleven) patient is not able to do this Delirium
Drugs (medications and illicit drugs)
FRONTAL LOBE COGNITIVE TEST E - Emotions (Anxiety, loneliness, nervousness)
- Verbal Fluency Test M - Memory (Benign Forgetfulness)
o How many items can you buy from the E - Endocrine (DM, Thyroid) Ears Eyes
supermarket and name them in one minute N - Nutritional (B12 Deficiency - Self-Neglect); Neurological
(Normal: >15; 15 or less is abnormal) problem (CVA, CVS)
o Say as many words as possible in one T- Tumors, Trauma (Head Injury)
minute starting with letter F, A, S (normal is I - Infection (HIV, Syphilis)
15 words/letter or 30 words in all three) A - Alcohol, Amenesia (WKS)
o Or name as many vegetables. fruits, or S - Chronic Schizophrenia
animals in one minute (10 or greater is
normal) Common causes of dementia:
- Interpretation of Proverbs - Alzheimer - 60 % of dementias
o A stitch in time saves nine - Frontal Lobe - 10%
o Time and tide wait for none - Lewy Body Dementia - 10%
- Similarities and Differences: What's the difference - Alcohol - 5%
between a bird and plane or table and chair - Vascular - 15%
- Motor Sequencing: Fist edge palm or rapidly
alternating movements History
- Can you tell me more about it? (My family is
Patients with frontal lobe dysfunction perform poorly and concerned that I am getting forgetful and that I am
disorganized. afraid I have dementia.) Do you forget about the
recent things or the past events? Do you misplace
Diagnosis and Management items like car keys or keep the stove open? Any
problem in performing daily activities? Do you find
- Based on the history and assessment, my diagnosis is difficulty in planning and decision making? Do you find
frontal lobe dementia. This is because of the following it difficult to remember the names of your friends and
reasons: family? Did you notice any change in your
o History given by his wife (irritability and personality? (like getting irritable or not being yourself
personality changes) which shows Frontal lobe dementia) Any episode of confusion?
personality and behavioral changes (delirium)
suggestive of frontal lobe dementia - How is situation at home? Do you find it difficult to get
o MMSE reveals problems with registration along with family? Any problems or issues at work?
and recall and cannot do a simple task Any difficulty in finding your way back home?
which show problem in executive functioning (tendency to wander) Any recent accidents? How
o My history shows that the patient had much is this affecting your personal and social life?
problem with understanding and severe - Do you have any headaches? Any previous head
short-term memory loss (forgets questions injury? episodes of falls? Problem with hearing?
and doesn't answer questions correctly), Vision? Chest pain? Shortness of breath? What about
lack of insight and cognitive impairment your diet? Any problems with waterworks? Bowel
o Specific Frontal Lobe Tests are poorly done. movements?
- How is your mood? Do you enjoy the things that you
- I would like to do investigations and look for reversible used to enjoy? Do you have any weather preference?
causes. Secondly, I will refer this patient to a Past history of Heart Attack? Stroke? Mental Illness?
psychiatrist for full medical assessment and Infections like HIV or Syphilis? Are you on any
neuropsychiatric testing. On MRI, Frontal lobe atrophy medications?
can be seen. - Family history of Mental illness or dementia and other
- There is no know curative treatment and supportive serious conditions? SADMA?
care is essential. This condition is managed by - Is there anybody at home to take care of you?
multidisciplinary care team including support groups.
Median Survival time is seven years and often occurs Diagnosis and Management
between 40s and 50s. - Dementia is only a diagnosis of exclusion and you
- In Frontal lobe dementia, earliest manifestations are need complete medical evaluation and further
personality changes and alterations of behavior neuropsychological testing by a specialist.
including social dysfunction. - I will order the investigations and once the results are
back, I will refer you to the specialist
Forgetfulness in a 56-year old man (Case 111) - Give the reading materials.
- Once the results are back, refer the patient.
Case: A 56-year-old man comes in your GP clinic with complaint - Red flags. Be careful of driving and avoid any falls.
of forgetfulness but otherwise he is healthy. Report any accidents at home.

TASKS: Investigations
a. Perform a history - FBE, LFTs, UEC, BSL, URine MCS, TFTs, BGL
b. Do MMSE - Vit B12 and Folate, Vitamin D, Calcium and
c. Explain to the patient the results and further Phosphate,
management - Syphilis and HIV (with patients consent)
- CT scan or preferably, MRI of the patient
6

Dementia and Disclosure of patients condition (Book case 119 b. Physical examination/investigation results (FBE 140,
pg. 641) MCV 107, Plt 300, LFTs GGT increased, other
enzymes normal including albumin, RFTs normal, BP
Case: Michael aged 70 years had come to see you in your GP 150/100mmHg,
clinic. He is concerned about his wife Jenny who had increasing c. Management advice
forgetfulness over the past 6-12 months. She has misplaced her
bag and bank cards on numerous occasions. She is spending History:
very little time reading or knitting which were her favorite - Establish pattern of drinking: I know you are
hobbies. You had seen Jenny last week with URTI. Michael is concerned about your drinking. It is a very good
interested that if you can recommend Jenny for Nursing home decision to come and see me. I need to ask several
placement and is requesting for your approval letter. questions that may be personal. Is it alright? Since
how many years have you been drinking (years)? How
Task much do you drink per week? What type of alcohol do
a. Focused history you drink (spirit, beer, wine)?
b. Management advice o Safe drinking: 1 SD for female, 2SD for
males per day everyday
- Assess MMSE - Where do you prefer to drink? With family, friends? Is
- Draw clock test: it binge drinking or continuous? Are you aware of safe
o Circle: 3 points level of drinking? Have you noticed any ill effects of
o Number of the clock: 2 alcohol on you? Do you think you can drink heavily
o Right numbers: 2 without appearing drunk (tolerance)? Are you able to
o Put the time of the clock: 2 work as efficiently? How is it affecting your
- Investigations/Screening Test: relationships at work and in home? Have you ever had
o RFTs, LFTs, TFTs, FBE, Blood glucose, any accidents related to alcohol?
serum electrolytes, calcium and phosphage, - CAGE:
urinalysis, serum vitamin B12 and folate, o Have you ever thought of cutting down?
serum vitamin D, syphilis serology (HIV), o Do you feel annoyed when people criticize
CXR, CT/MRI, you?
o PET or SPECT scan for further information o Do you feel guilty for taking alcohol?
- Multidisciplinary assessment (aged care assessment o Do you take alcohol first thing in the
team/memory clinic): geriatrician, occupational morning?
therapist, psychologist, etc - How motivated are you on a scale of 1-10 to quit/cut
down on your alcohol?
Features Dementia Pseudodementia - Withdrawal Effects
Onset Insidious Clear-cut, often acute o How long can you go without alcohol? Not
Course over 24 Worse in evening or Worse in morning more than 1 day
hours night (sundown o How do you feel after a period of
effect)
abstinence?
Insight Nil Present
o Do you think you need to drink to sleep?
Orientation Poor Reasonable
Memory Loss Recent > remote Recent = remote
- Social effects
Responses to Agitated Gives up easily o Have you noticed any problems at work with
mistakes alcohol?
Response to Near-miss! Difficulty dont know; slow o How is your relationship with partner and
cognitive testing understanding and reluctant but children?
(question) understands words (if o How is your financial situation?
cooperative) o Have you had any accidents/fights because
of drinking
DRUGS, SUBSTANCES OF ABUSE AND ALCOHOL - Health problems
o Have you ever noticed heartburn, gastritis,
Alcoholic Counseling heart disease, liver disease, anemia,
hypertension, problem with memory, mood
Case: You are a GP and a 47-year-old businessman comes to changes, depression, change in sexual
you to discuss his alcohol consumption because he got pulled performance (thought it was related to age)
over by the police on his way to work. The blood alcohol level - SADMA
was 0.04. He was given a warning as it was near the legal limit - I will need to organize some investigations to see
and a sign that he had a lot of alcohol last night. He wants to effect of alcohol. FBE, anemia (macrocytic-vitamin
discuss the safe level of alcohol and the effect of alcohol on a b12), LFTs, Lipid profile, serum lipase, BSL, liver
person. USD, ECG,
- 1 SD = increase blood alcohol concentration by 0.01
Case 2: Jarrod aged 30 years is a new patient to your clinic. - Liver takes 1 hour to metabolize 1sd
Jarrod states that he has been drinking on average four SD per
day per week for the last six months since starting his new job. Counseling
Before this he was consuming on average 2SD drink two days - Feedback: History shows that you have been drinking
per week. Last night while drunk, he met a minor accident and more than normal. This is why I ordered some tests to
his girlfriend asked him to see you as she is not happy with his determine the effect of alcohol in your body. The high
drinking habit. Jarrod works in a local supermarket and is level of alcohol may cause HTN, cause tummy
otherwise fit and healthy. He is not on any regular medications problems (heartburn), increased weight, affects your
and had no known allergies. liver, heart, brain, loss of memory, gout, sexual and
social problem
Task - Listening: What do you think?
a. Further history - Aim for safe level of drinking: Advise on safe level of
drinking (1 30 ml spirit = 1SD; restaurant wine =
1.8SD)
7

- Goals (short-term) - Provide regular feedback regarding the impact of


- Strategy: alcohol upon their physical, mental and social health
o Dont drink daily - Minimize the harms from polydrug use, by advising
o Drink only with food against and offering treatment for other drug problems
o Have a glass of water between drinks to - Monitor prescribed and complementary medications to
satisfy your thirst avoid predictable drug/alcohol interactions. Identify
o Switch to low alcohol drinks and respond to problems of poor medications
o Mix alcoholic drinks with non-alcoholic ones adherence in heavy drinkers
o Always check bottle for SD - Use strategies to enhance patient engagement,
o Avoid high-risk situations (with alcoholic including approaches to overcome barriers posed by
friends, going to the pub) cognitive disorders, language, and cultural issues or
o If you are under pressure, tell them my physical disabilities
doctor told me to cut down - Define and attend to any specific medical and
o When youre stressed, take a walk; explore psychiatric conditions with relevant services that
new interests, plan new activities communicate regularly
o Start with a period of abstinence to test the - Consider strategies to minimize the consequences of
presence of withdrawal symptoms. If you specific medical complications such as CNS and
want to drink, please report right away. We peripheral nerve damage, liver disease, cerebellar
can manage your symptoms damage, and/or peripheral neuropathy
- If you consider cutting down to safe levels or quitting - Engage psychosocial supports (meals on wheels,
alcohol, it will have a positive effect on your health, welfare, employment support, community and
save money, have less family problems, and more religious networks, financial or relationship
time to spend with your family. I would also like to counseling) to reduce personal and family harms
recommend for you to join alcohol anonymous which - Empower family and close friends to reduce
is a support group and recommend lifestyle availability of alcohol to encourage further
modification for weight reduction and control engagement with clinicians able to help with alcohol
hypertension, but it cannot be successfully done until problems
alcohol is taken cared of. Im always there with you to - Consider any medico-legal or ethical obligations,
help you and support you, but in the end, the decision including driving assessment, child protection, welfare,
is yours. guardianship and employment issues for use in
- Offer to arrange for family meeting to discuss about certain trades or professions
alcoholism
- Review once the tests come back Smoking Counseling
- Refer to DETOX unit if dependent! Alcohol withdrawal
scale: diazepam Case: You are a GP and your next patient is a 30-year-old
- Red flags: Any other major concerns female. She has been recently discharged from the hospital due
to recurrent attacks of bronchitis. She smokes 1 pack of
Binge Drinking cigarettes per day since the age of 18. A few weeks earlier, you
saw her and advised her to stop talking. Now, she would like to
Case: One of your patients David who is a single parent brought quit.
his 10 years old son Simon to see you who got sprained ankle.
You examined his son and diagnosed him as having a sprained Task
ankle. On examination, apart from the sprained ankle, there Is a. Counsel her regarding smoking cessation
no other injury, bruises, or scars. The childs mother has left a
few years ago and the child was looked after by his father. - You made a very good decision to come here to quit
smoking.
You have seen David 4 weeks ago in your clinic due to minor
head injury after he got drunk and fell in the pub. At that time, Assess the Motivation
you noticed he is a binge drinker of 24 pints on every weekend. - How motivated are you to stop smoking on a scale of
You told him about his overdrinking, and you asked him to come 1-10? How confident are you that you will succeed (1-
back in 3 weeks time to discuss the issue, but he did not show 10)?
up on his appointment day. Today, he is here for his sons sake.
David is a delivery driver. Assess Dependency
- How many cigarettes a day do you smoke? How soon
Task after you wake up do you light your first cigarette (if
a. Talk to the father about your concern within 30 minutes of waking up high chances of
giving NRT, bupoprion or champix)? Do you find it
Strategies for Working with First Presentation of Alcohol Abuse difficult not to smoke in a non-smoking area? Is it the
first cigarette that is hard to give up?
FLAGS Approach
- Feedback: tell patient your impression about his intake - What is the pattern (smoke it with friends or out on a
level party or by self)? Do you smoke even when you are
- Listen to his reaction very ill? Have you tried to quit smoking before? If yes,
- Advice about the benefits of quitting why did you fail?
- Goals setting: keep it with safe limits or stop
- Strategies: quench thirst with non-alcoholic drinks Advice on Nicotine Withdrawal
before having an alcoholic one, avoid drinking on an - Within 24 hours of stopping or reducing nicotine, you
empty stomach, switch to low-alcohol beer, think of a may experience some symptoms such as depression,
good explanation for cutting down on your drinking insomnia, restlessness, irritability, anxiety, difficulty in
concentrating, drop in heart rate, increased appetite,
Strategies for Working with Persistent Problem Drinker craving for sweets and cigarettes, but these symptoms
- Continue to encourage a reduction or cessation of peak over a few days and will resolve after about a
alcohol intake month.
8

Benefits of quitting of Smoking - Reading material.


- Start to smell better - If partner is smoker, ask patient to come and see you.
- Food tastes better - Quit Line number.
- Circulation of the blood improves - Review after 1 week and see progress.
- Better immunity and less sick days at work - Support groups.
- Save money (2500 dollars/year for pack-a-day
smoker) Request for a Repeat Prescription of benzodiazepine (Drug
- More time to spend with family Dependence)
- Set positive example for children
- Minimize risk for heart disease, lung cancer, stroke, Case: Michael aged 22 years comes into your practice with no
and gangrene appointment, seeking oxazepam to clam him down and sleep.
He looks a little unkempt, is agitated, fidgets and shakes his
Plan legs while seated. The reception staff reports feeling
- Best way is COLD TURKEY. Decide the date to stop uncomfortable around him and note that he was insistent to be
smoking within 2 weeks of making a decision. seen today. There was no overt aggression. When you ask
- Aim for TOTAL ABSTINENCE and not just cutting about history of benzodiazepine use, Michael becomes irritated
down. and says he is asking for one script, but says he is been using
- Review your previous attempts at quitting and what them for years, started when he was having difficulty coming off
went wrong. speed and the he had then become dependent. He says his
- Inform family, friends, and other smokers about your current supply was stolen with car this morning which has left
plan him in anxious state with nothing to calm him down. He says he
- Avoid alcohol and review coffee intake (triggers) usually gets them from a GP on the other side of the town but
- To decrease cravings, drink plenty of water, gradually unable to get there without car. He says he uses 5-6 tablets of
increase other activities, avoid situations which could 30mg oxazepam per day
restart your smoking, and eat more citrus fruits Task
(vitamin C helps reduce cravings). a. How will you address the request of the patient
- Nicotine replacement therapy - I am ready to help you. That is why I am here.
o Contraindications: pregnancy and CAD Because you are not my regular patient, I need to ask
o Nicotine patches: 40% nicotine; no tar and a few more questions before I can help you.
other carcinogens; - Why are you taking this medication? How long have
o Nicotine gums (2 or 4 mg): chewed you been using this medication? How many times do
intermittently for up to 30 minutes 10x a day; you take this in a day? Did you increase the doses by
poorly absorbed in acidic environment so yourself?
decrease fruit juices when youre chewing - Drinking, smoking and illicit drug use?
gum; can also cause mouth soreness and - General health? serious illnesses, injuries or accidents
dyspepsia (car accident)? Fits, falls, faints or loss of
o Transdermal patches (7 -21 mg used 16 0r consciousness? Did you ever feel depressed or
24 hours):applied every morning on a diagnosed with depression or other psychiatric illness.
rotational basis to non-hairy areas; can lead
to skin rash Management
o Nicotine inhalers (4mg): single use only; 10 - Explain: period of monitoring your use and moods or
or more per day for 6 months stress followed by graded reduction, along with
o All are equally effective. Monotherapy is regular appointments, support and resource materials
preferred. or groups
- Bupoprion (Zyban): atypical antidepressant with both - Diazepam (valium): long-acting I will change the
noradrenergic and dopaminergic activity. It works on medication to the same medication but will work
addiction pathways; 150 mg per day for 3 days then longer. Pick up daily supply.
increase to BD x 9 weeks. SE: nausea, insomnia, dry
mouth; CI: epilepsy, diabetes, pregnancy Morphine Request for Psychogenic Pain
- Varenicine (Champix): blocks the nicotine receptor
and dopamine (addiction) pathway as well; does not Case: You are working as night shift HMO in ED and your next
contain nicotine and is not addictive; only available on patient is a young lady with abdominal pain. Investigations were
prescription; CI: mental illness, kidney problems, done and were found to be normal. She is requesting morphine
pregnancy or breastfeeding; not yet studied on for her pain. This is not the first time she came with abdominal
epilepsy; if youre taking champix, not allowed to take pain.
NRT or Zyban. Start 1-2 weeks before quit date
because champix needs time to build up in the body Task
and to allow it to start working; can smoke while taking a. History (happening since 4 or 5 years;
champix but make sure to quit by the date set; comes b. Management
as a white (0.5mg) or blue tablet (1mg).
o Days 1-3 0.5mg OD History
o Days4-7:0.5mg BD - I understand that you have this tummy pain and the
o Week 2-4: 1mg BD (give prescription for 4 investigations we did came out to be normal. This
weeks) means that we could not find any organic cause for
o 5-12 weeks: 1mg BD (give prescription for 8 your pain. I also understand that this is not the first
the rest of 8 weeks) time. Since when is this happening?
o If patient has stopped smoking, recommend - Confidentiality
another 12 weeks of treatment for the long- - Do you think something happened in your life or a
term effect. Do not take double dose if you stressor that initiated it (happened after getting
missed the tablet for >6 hours. Dont share separated from partner)? How often does it happen?
medications. If you start smoking again, may Do you think these episodes are related to any
have nausea and vomiting. particular stress? Have you seen any doctor for this
9

condition (saw GP and gave pethidine)? Where do (persecution) do you think youre
you get the morphine from? Have you noticed any special? (grandeur) do you think
palpitation, agitation or sweating when youre not someone is putting thoughts on
taking morphine for your pain (if yes, means your mind? (control) do you think
addicted)? youve done something wrong?
- Depression questions: How is your mood? Do you still (guilt) do you think radio and TV
find things pleasurable? Any changes in your sleep, are talking about you? (idea of
appetite or weight? Do you think life is worth living? do reference)
you have any thoughts about hurting yourself or o Depression: how is your mood lately? Do
others? you enjoy the things you used to enjoy? Any
- Whom do you live with? Any stress at home or at problems with memory or concentration?
work? Any financial problems? Do you have enough Change in sleep, appetite or weight? Are
support? Do you have friends? SADMA? you interested in your sexual life? Do you
think life is worth living? Have you ever
Management thought of harming or killing yourself? Or
- Let me reassure you that I will try to manage your others? Have you ever tried this in the past?
pain. I understand that your pain is real. There is If you leave this room, what are you going to
something called brain-body axis. Anytime our mind is do?
stressed, our body starts reacting (e.g. diarrhea before o Anxiety: Do you feel nervous as a person
exams). In the same way, your body is reacting by most of the time? Tremors?
producing abdominal pain because of your stressors. Palpitation/pounding of the heart?
- Morphine is a short-term relief for the pain and has got - Social history
many side effects. It can affect your respiratory o Home situation: how are things going at
system, heart, and is highly addictive. At this stage, I home? Are you experiencing any problems?
will give you panadeine forte to start with and refer o Employment: do you work? Any problem at
you to the psychologist. He will do talk therapy (CBT) work? Any financial problems?
to relieve your stress and he will teach you how to o Social circle/friends?
overcome and handle your stress. If you need social o Hobbies? What do you do for relaxation?
support, I can organize a social worker. If you have - Past history: mental disorder? Depression?
financial issues, I can refer you to centerlink. There Psychoses? Medical illness? Thyroid problem?
are a lot of support groups available for you. You are Medications and side-effects?
not alone. - Family history: mental disorder? Thyroid?
- If dependent: Refer to psychiatrist for drug - SADMA!!!
dependence management
- Do you agree with me or do you have any other Mental Status Examination
questions? - Appearance: properly dressed? Unkempt?
Disheveled?
PSYCHOTIC DISORDERS - Behavior: cooperative/uncooperative; comfortable?
anxious? Restless? Irritable?
Paranoid Schizophrenia - Speech: coherent, fluent, understandable? High/low
volume? Monotonous/changing tones? Pressured
Case: A 35-years-old female is in your GP clinic and wants a speech?
letter to the Department of Housing Authority because she - Mood/affect (congruent)
wants to change her accommodation. She had schizophrenia for - Perception: hallucinations Psychosocial history
the last 10 years and she was on haloperidol. On examination, - Thought:
you can see some contact dermatitis on her hands. o Content: delusion/suicide
o Form: how contents of thought are
Task expressed (ie flight of ideas, loose
a. Psychosocial history (Neighbor wants to harm her and associations,tangentiality)
throws things into her home. She lives alone at Centre - Cognition (Orientation): Time? Date? Person?
link and believes the TV is talking about her. Didnt - Insight: do you think that you need help? Or medical
see the psychologist for 3 years and cut down dose by advice?
since 1 year. Wash excessively with cleaning - Judgment: what would you do if there is a fire in this
agents twice a day.) building?
b. Mental Status Examination (well-dressed, groomed,
mood/speech normal, delusions of reference, auditory
hallucination (hearing voices that neighbor is talking Differential diagnosis:
about her), delusion of persecution and husband is a. Organic causes (brain tumors)
involved, dermatitis (throw things at home), no insight, b. Drug-induced or substance abuse
good judgment, oriented, no suicidal ideation, stooped c. Anxiety disorder
medication by herself she thinks she's feeling well) d. OCD highly unlikely
c. Give your findings to the examiner
d. Diagnosis and differential diagnosis Management:
- I will need to refer her to the hospital because she is
- ENSURE Confidentiality! living alone, has ideas of reference, paranoid
- Psychosocial history: HEADSSS delusions and is lacking insight. I will need to contact
o Psychological: the specialist at the hospital to review her. This will be
Auditory/visual hallucinations: do for her safety.
you see or hear things that - Urgent referral to psychiatrist for possible admission: -
nobody else can see/hear? due to loss of insight, paranoid ideation, not taking
Delusions: do you think somebody medication, living by herself;
wants to harm you? Following - If patient refuses: involuntary admission
you? Spying on you?
10

Relapse of schizophrenia (Tardive Dsykinesia) ideas that are contrary to fact. There are many causes
for this condition. Schizophrenia which needs around
Case: A 40 year old lady with schizophrenia for the last 15 years 6 months to be diagnosed or schizophreniform or
comes to your GP clinic because she has movements of her delusional disorder. Some patients may have medical
face. conditions which we call organic-induced psychosis.
Others may use illicit drugs which can experience it as
Task sequelae. We call this drug-induced psychosis. If any
a. Psychosocial history patient is diagnosed with acute psychosis, it is an
b. Diagnosis emergency situation as the patient is not safe for
c. Management himself or for people surrounding him. They may have
suicidal ideation or any psychotic ideation which could
History make them very aggressive and harmful to
- Ensure confidentiality themselves or others. Under the mental act, we
- Tardive dyskinesia-- more with typical antipsychotics usually admit involuntarily all acutely psychotic
- Can you stick out your tongue for me? patients until we stabilize their condition and we do
- Side effects: postural hypotension (giddiness, light further assessment to find out the cause. Usually,
headedness with posture change),dry mouth? Bov? many persons share the management of those
Urinary retention? Constipation? Milky discharge on patients for short-term and long-term management.
breasts? Loss of libido? Decreased sexual drive? Family will be notified and a meeting will be done to
Problem with periods discuss the management. We need lots of support
- EPSE: stiffness? Restlessness? Gait problems? from you. The psychologist, psychiatrist, mental health
Bradykinesia? Cogwheel rigidity? Tremors? nurse, social workers can be part of this team as well
- previous history of NMS? Fever, stiffness, confusion as myself as your GP. You can go to the hospital with
- Medication: are you taking the drug? Did you change your son and you will be notified with the further steps.
your dose? When did you see your psychiatrist? - Do you think my son is using illicit drugs? As I have
- Do psychosocial history? mentioned before, there are many causes. We need
- Do you think you need medical help? to assess him first. Our first priority is to stabilize your
son, then find out the cause and you will be informed
Management accordingly.
- Refer back to psychiatrist. Stop drug and change to
other medications. Ice-Induced Psychosis
- Risk of breakthrough psychosis
- May consider admission Case: A 20-year-old man was brought to ED by his friends
where youre working as HMO. He had hallucinations and
Drug-Induced Psychosis delusions. He was aggressive and violent. You sedated him with
medications. He went to a party last night and you suspected he
Case: An 18-year-old male who failed in one of his exams came used ICE. His father is here to see you. He knows about his
to your GP clinic for consultation. He felt depressed since then sons ICE usage.
and suffered insomnia. He came requesting for sleeping pills.
On assessment, you detected that he is suffering from Task
delusions, hallucinations, and other symptoms upon which you a. Relevant history
settled with the diagnosis of acute psychosis. He also admitted b. Advise further management
the use of illicit drugs. His parent came to the clinic to discuss c. Answer his questions
his case. The patient gave permission to discuss his case but
not to disclose his illicit drug use. History
- I understand you are here to talk about your son. Let
Task me assure you that he is in safe hands. Before I
a. Talk to the father explain the further management to you can you share
b. Explain current situation what you know about his condition? OR Do you know
c. Answer his question what happened in the party? OR It can be quite
common at a young age and I understand that John
Questions: has a problem related to this. may I know a bit more
- Is my son using drugs? about it?
- Is this condition due to depression? Can I take him - Since when is he using it? any previous hospitalization
home? like this because of this? Any intervention done or
- His auntie had schizophrenia, does he have this also? step taken regarding this issue? Are you a happy
Can he develop it? family? How is your family life? Any particular issue?
Does he have any siblings? How is his relationship
Counseling with them? Anybody else in the family using drugs?
- Let me assure you that your son at the moment is How much is this affecting the family? Does he go to
having a thorough assessment as we found him school or uni? How is his performance over there?
suffering from a sort of psychiatric emergency that we Any problems at uni or work? Any problems with the
call acute psychosis. I have contacted a team called law? Any of his friends having similar problems? Any
CAT whos undertaking the assessment. This team is other hobbies or sports?
the crisis assessment team. Psychosis is not a - How is his mood most of the time during the day?
specific disorder. This is a condition where a patient Does he enjoy the things he used to enjoy? What
has severe impaired sense of reality with emotional about his sleep? Did you notice any changes in his
and cognitive disabilities. The patient talks and acts in weight or appetite? Did he have any previous attempts
a bizarre fashion and may suffer from hallucinations to harm himself or somebody else? Did he ever talk
wherein he can see or listen to voices or things which about seeing/hearing things/voices that nobody does?
are not real or cannot be experienced by others Has he exhibited any strange behavior?
around. Also, he can suffer from delusions which are
11

- Any previous medical problems such as thyroid not worth living? Does she feel guilty about anything?
disease? Does he take any medications? Any Any time that her mood is really high?
allergies? FHx of similar problems and psychiatric - Psychotic: does she hear/see things that others do
illnesses? Smoking? Alcohol? not? Does she have any strange feelings or
experiences? Does she tell you that somebody is
Management putting ideas on her head or that the TV or radio is
- Most likely the condition that he is having is called ice- talking about her? Does she have strange
induced psychosis. Psychosis is loss of contact with experiences or abnormal thoughts? Does she think
reality that usually includes hearing/seeing things that there might be something wrong with her?
are not there and having abnormal beliefs. These are - How is her general health? Has she been diagnosed
called hallucinations and delusions. In simple words, it with any mental illness before? Family? SADMA?
is the changed and different way of thinking, speaking - I can see that you are tired. Do you have enough
and behaving that can make a person aggressive and support? Whom do you live with? Are there any
violent and unaware of his surroundings. This is what financial problems at home?
John is going through at the moment and this is
because of his ice usage. It can change the chemicals Management
in the brain to produce these effects. Now he is safe - From the discussion we have, your wife might be
and stable. suffering from a condition called postpartum
- He will be assessed by the CAT for psychiatric psychosis. It is not an uncommon condition but it
assessment. He will be admitted in the hospital under needs to be treated urgently. I am concerned about
care and supervision because in this condition he can you and your babys safety. I will need to admit your
harm himself and others. Even if the patient refuses, wife and I will call the psychiatric registrar to come and
they can be admitted involuntarily and it is in the best take a look. She will also be seen by the consultant.
interest of their safety. They can give him - At this stage, they might start with ECT and
antipsychotic medications for short-term to treat his antipsychotic medications.
intoxication. He will also undergo some investigations - How is your mood? Are you alright? I can organize a
such as FBE, U&E, LFTs, RFTs, BSL, TFTs including social worker for you.
urine and blood drug screen, alcohol concentration - Centerlink for financial problems.
and CT scan to r/o any organic cause. - I dont think she will agree to be admitted. I am sorry
- Once discharged, he will be followed up by but she will be admitted involuntarily under the mental
psychiatrist and GP. He can also be referred to a drug health act and I will call on the crisis assessment
rehabilitation center to help him stop drug usage and team.
develop new coping skills that make the relapse less - Dont worry. We will be here to take care of her.
likely. Prognosis is good. If you need any help or you have
- Arrange family meeting. any other concerns, please dont hesitate to contact
- Support groups. us.
- Refer to psychologist if father is depressed.
- Is there any antidote available? No. MOOD DISORDERS
- Is ice addictive? It is a highly purified form of
amphetamine and thats why it is powerfully addictive. Loneliness or Empty Nest Syndrome
- Will this lead to schizophrenia? Using this drug is a
risk factor for mental disorders. Case: A middle-aged lady presented in your general practice.
- Will you report to authorities if drug test is positive? It She complains of feeling down and depressed for a few months.
is a confidential issue. Once he is stable, we will talk You asked her to come for consultation a few weeks back but
to him and discuss further plan of action with him. did not come.

Postpartum Psychosis Task


a. History (poor appetite, lack of sleep, early morning
Case: You are an HMO and your next patient in ED is the awakening, do not socialize, appetite okay but does
husband of a 25-year-old lady who had her first baby 2 weeks not find it pleasurable, no friends, dont feel like talking
ago. He is concerned about his wifes behavior but the patient is to friends, stay-at-home, separated with husband but
not concerned about her problem. The husband has got the not talking)
consent to talk about his wife. Delivery was NSVD. She did not b. Provisional diagnosis
want to come and see the doctor, so the husband came to talk c. Management
to you while waiting at the waiting room with the baby and the
nurse. Differential diagnosis
- Loneliness
Task - Depression
a. History (doesnt take care of the baby and husband - Adjustment disorder
too tired to take care; scared to leave baby with her; - Anxiety
behavior is odd; doesnt sleep much; lost weight, feels - Bipolar disorder (depressive episode)
like theyre both devils) - Organic (menopause, hypothyroidism)
b. Management (immediate and long-term management)
History
History - Confidentiality
- Confidentiality - How is your mood? Do you have low mood most of
- Could you tell me what you meant by change in the day? Do you still find the things you do
behavior in your wife? Is she breastfeeding at all? Is pleasurable? How is your sleep? Appetite? Weight?
she taking care of the child? How is your energy level? Do you think life is worth
- Mood: How would you define her mood? How about living? Have you ever thought of harming yourself
her sleep? Appetite? Weight? Do you think shes - Who are you living with? Do you talk to each other?
weak and has no energy? Does she think that life is How about your kids? Family?
12

- Are you sexually active? Stable partner? What do you - Sleep problem? How is the problem? Hard to initiate
do? or wake up early? Night sweat? Do you feel fresh in
- Do you hear or see things which others do not? Do the morning? Any nap during the day
you have strange experiences? - How is your mood? Appetite? Daily enjoyment as
- How is your general health? Do you have weather usual? Do you feel active or lethargic? Suicidal
preferences? Do you have swelling all over the body? ideation? Harming yourself or others? Do you think life
Weight gain? Lump in the neck? is worth living?
- Menopausal symptoms: irritability? Dryness of - Do you ever feel or hear things that other people
vagina? Hot flushes? Mood swings? Pap smear? cannot? Do you feel someone is spying on you?
Mammography? SADMA? - Whom do you live with at home? How is your
relationship with your family? Since your fathers
Diagnosis death, have you talked to someone else about your
- From the discussion we have, you most likely have a feeling
condition called loneliness or empty nest syndrome as - SADMA?
you have no one to talk to at home, your husband is - Tea and coffee drinking habit during in the evening?
estranged to you and your children have grown up - General health? Past history of thyroid problem or any
and moved out. These are all contributing to it. There mental illness? Any family history of similar problems?
are a lot of things we can do about it. You can join the Any family history of mental illness?
community clubs, or do voluntary work. Meet and - Insight and reliability
make new friends and create a social circle. You can
explore your interests and activities. Management
- I can arrange a social worker if you need a help. I will - The most likely diagnosis at this stage is one of the
refer you to a counselor with whom you can share and normal emotional reactions to people who lost
talk about things. If you agree, I am happy to organize someone who is very close and emotionally bound. It
a family meeting and tell them about your condition. is normal to feel disbelief, anger, sadness.
You can always give them a ring or talk to them via - However, I can help you with some advice in many
skype to see them. ways.
- Lifestyle modification. Review. o Socialize more talk to friends and family
- Referral to psychologist. Reading material (Beyond o Approach religious resources according to
Blue). your beliefs to help your relax spiritually
o I can also organize a support group for you
Normal Grief and your family
o Sleep problems provide with written
Case: You are a GP and 18 years old university student comes materials regarding sleep hygiene and other
to you with complaint of poor sleep since her father died. She techniques
cant concentrate on her study and she is anxious as the exam Avoid having tea or coffee in the
is approaching. She visited you 2 months ago with some flu. evening
She was alright at that time. Avoid having heavy meal before
sleep
Tasks A glass of warm milk before sleep
- Focused history Try to maintain the room
- Management environment being not too hot and
not too cool
Try to sleep in dark and quiet
room
Have a routine to go to bed at the
same time everyday
Avoid day time naps
Meditation before bedtime can
help you relax
I can arrange a referral letter to
psychotherapist who will teach
you relaxation technique.
- I can also organize social workers to visit you at your
place as required. It will be difficult for you to go to
exam right now, so a letter will be provided to your
principal of your school to reschedule your exam
- University counselor is also available for counseling of
Stages of Grief: (normal grief can go up to 3 months) such cases
- Shock and disbelief o Sleep hygiene and life style modification
- Grief, anger, despair, self-blame, guilt o Prescription short acting benzodiazepine
- Adaptation and acceptance of the loss o If you feel very low at any time and you feel
stressed and frustrated with yourself, please
If the timing and severity increase, there is high risk of suicide come to me and contact crisis control
and psychosis. center.
o Please do not stay alone and I will review
History you in 3 days time about your progress
- Sorry to hear what has happened. How are you
coping with this situation and your family? Is there
anything you want to share with me regarding your
dad?
- Confidentiality!
13

Anniversary Grief Reaction SSRIs for some time. She did not show up for previous follow-
ups for the last 2 months. She is here today because the
Case: Your next patient in GP practice is a middle aged woman receptionist has called her.
who came for regular checkup regarding her BP. She had no
emotional problems before but during the last weeks, she was Task
tearful and often crying. Her husband died of heart attack 12 a. Perform Mental state examination
months ago. b. Tell examiner diagnosis and management plan

Task Criteria:
a. History (started 2 weeks ago, when I was cleaning the - Anhedonia, depressed mood, suicidal ideation, sleep
closet and putting his clothes aside, and started to problems (early awakenings), lack of energy,
smell his scent; I can feel his presence) problems with concentration and decision making,
b. Diagnosis and Management lack of sexual desire and appetite
- Diagnosis depends upon the presence of 2 of the
History above along with suicidal ideation, persisting for at
- I understand that you came to see me for review of least 2 weeks or any 4 of the above without suicidal
your blood pressure. Is everything alright? Have you ideation.
been checking your blood pressure? I also understand - Risks/criteria for admission: not eating/drinking
that you have been tearful and crying. How do you appropriately, suicidal ideation, lack of support at
feel right now? (Patient starts crying Offer tissue and home, not taking/responding to antidepressants
Water). I know it is a very hard time for you. I am here
to help you. If you feel like talking to me, let me Counseling
reassure you that everything we talk about it - Show empathy. Confidentiality statement.
confidential. I will not breach this confidentiality. - From the notes I understand that you have been upset
- When did it start exactly? How did you cope after your since the incident five months ago. Can you please tell
husbands death? Hows your mood? Do you still find me exactly what happened? I understand it is very
things pleasurable? Hows your sleep? Appetite? difficult for you to go through that experience one
Weight? Psychomotor retardation or agitation? Do you more time, but it will really help me to understand the
think life is worth living? Do you feel guilty about your situation. When exactly did you start feeling bad about
husbands death? Have you thought of harming yourself? How was your mood before the incident?
yourself or anybody else? Do you hear or see things Were you eating and drinking well? Were you able to
that others do not? Do you have any strange work? Have you ever been diagnosed with depression
experiences? or other illnesses like thyroid problems, diabetes,
- Whom do you live with? Have you got enough support infections? What happened after the incident? Did you
from friends and family? Do you go out with friends? notice any changes in your weight or appetite? Were
Are you working at the moment? Can you do your you feeling guilty all the time? Any change in your
day-to-day activities? SADMA? sleep pattern? Any early morning awakenings? Did
you feel like harming yourself or others? Do you think
Diagnosis and Management your life is worth living? Have you thought about how
- From the history, most likely what you are you are going to do it? Any plans? Did you buy
experiencing is anniversary grief reaction. This is something for that plan? Please tell me, whom do you
normal, expected and understandable especially when live with him at home? Any partner? Kids? Relatives?
a close person/loved one passed away. Your mind Friends? Neighbors to take care of you? Are you
ventilates the feeling through crying. To feel your working at the moment? When did you leave? Can
husbands presence is a part of anniversary reaction you tell me more about the medications that were
and it doesnt mean that you are getting insane. I given to you? How long did you take them? Did they
understand that you are going through a tough time. help to improve your mood? Why did you stop?
What you are feeling is like a bruise. It will heal - Do you see/hear things that others dont? Do you
without scarring. You will feel better once the have strange experiences? Do you think some people
anniversary phase is better. But what you need at this are trying to harm or spy on you? Are there repetitive
time is emotional support. We will manage your thoughts that you cant get rid of? Do you think the TV
condition with a multi-disciplinary approach or radio talk to you?
(Psychiatrist, Psychologist, Occupational Therapist, - Can you please tell me your date of birth? Day?
Social Worker, Counselors, and Mental Health - What would you do if there is fire in the room or
Nurses). I will refer you to the psychologist whom you envelope on the street that has an address on it?
can share your problems with and to help you cope - Do you think you need medical help? May I ask why
with the grief, social worker, and grief support group. youre here?
If you are happy, I can arrange a family meeting. - What are your plans for the future? Are you planning
- Cant you just give me medications doctor? You do on anything?
not need any medications at this moment. All you
need is a lot of support during this hard time. MSE (ASEPTIC)
- I will need to see you in a weeks time to see your - Appearance (dress, posture, hygiene)
progress. - Speech (rate, tone, volume)
- Referral. Review. - Emotion (affect and mood)
- Perception (hallucination, illusion, derealization)
Major Depression with Psychotic Features - Thought (delusions, suicidal/homicidal ideations,
obsessions, logical/coherent)
Case: You are a GP and a 42-year-old nurse comes to see you. - Insight and Judgment
She had been accused of an incident at the hospital around 5 - Cognition (orientation to time, place, and person;
months ago where a patient had died. The nurse has been memory; LOC)
cleared by the coroner and the case was adjourned. The patient
did not feel well after the incident and she was treated with
14

- I would like to address my MS findings to the - Youre doing a good job as a mother. Dont worry. I do
examiner. The patient looks appropriately dressed for understand that it is difficult to be a mother for a first
the weather. She looks gloomy, tearful, and avoiding time and you need support. I will organize a social
eye contact. She is sitting with a drooping posture. worker to help you. If you like, I can organize a family
The affect appears constricted, although the mood is meeting and talk to your husband about the issue. I
depressed and irritable at times. The patient speaks would also like to refer you to a counselor to teach you
with a monotonous voice, sometimes with long pauses how to cope with stress.
in between where she avoids answering. I also found - I would organize basic investigations especially FBE,
that the patient has delusions of guilt. She feels ESR/CRP, urine MCS, BSL, and TFTs.
helpless and has suicidal ideations although no - Do not worry. You are not alone. These blues or mood
particular plan is present at the moment. Her cognition swings should be fine in around 1-2 weeks (1 month
is distracted where the patient is not able to maximum).
concentrate adequately although her memory is intact.
Her insight and judgment is impaired. Postnatal depression with psychosis/melancholic features
- Based on the examination findings my most likely
diagnosis is major depression with psychotic features. Case: 30-year-old woman came to your GP clinic. She has 2
It is obvious that the patient is neglecting herself. She children 30 months and 2 months. She presented with 2 weeks
needs to be evaluated appropriately by the psychiatric history of tiredness, weight loss, and inability to sleep. Shes
team so I will need to refer her to the hospital if always worried about her baby as she thinks baby will die from
required under the mental health act. The most likely SIDS. You arranged some investigations for her 1 week ago and
management is anti-depressants with or without ECT all the tests are normal. Today, shes here to collect the report.
followed by CBT later on.
Task:
Postpartum/Postnatal Blues a. History
b. Diagnosis
Case: Your next patient in GP practice is a 25-year-old Jane c. Management
who is 7 days postpartum. She feels exhausted, and has lack of
energy, and gets quite irritable at times. She is wondering if she History
is lacking some vitamins and seeks your advice. - I understand from your notes that you are here
because you have trouble sleeping, has lost weight
Task and are always tired? Can you tell me more about it?
a. History (1st baby, feels very tired; planned pregnancy; Can you describe me your sleep pattern? I know you
takes care of the baby; complicated labor prolonged are tired (anemia, chronic illnesses, psychological),
for 14 hours, eclampsia; cannot sleep at night but do you have any SOB, palpitations, fever?
because baby is crying all the time; husband needs to - How is your mood? Sleep? Weight? Appetite? Have
travel a lot; needs help; I love my baby; no past you lost interest in the activity which used to be
history of depression) pleasurable before? Do you think life is worth living?
b. Diagnosis Have you thought of harming yourself or anybody
c. Management else? Have you ever thought of harming your baby?
- Psychosis: do you see, hear, feel things which others
Risk factors do not? Do you have any strange experiences? Do
- Prolonged or difficult labor you think someone is putting thoughts into your head?
- First baby Or think something/someone is after your thoughts?
Do you think tv/radio/newspaper is talking about you?
History Do you think youre a good mother?
- Congratulations! How was the pregnancy? How was - Insight
the labor? Is it your first baby? Is everything okay - Judgment
now? How is the baby? Did you start breastfeeding? - Cognition
Any problems with that? - HEADSSS
- I understand that you have tiredness and youre - PMHx/FHx/SADMA
irritable?
- Confidentiality Risk factors for postpartum depression
- Any SOB or did you have a lot of blood loss? Do you - Previous history of postnatal depression
think youre pale? Any weather preferences? How are - Previous history of any mental illness
your waterworks? Hows your discharge? Any - Unplanned pregnancy
offensive smell? Hows your diet? - Difficult marriage/lack of support
- Mood: How is your mood? Do you still find things - Social isolation
pleasurable? Hows your weight? Appetite? Sleep? - Complication during pregnancy
Have you ever thought of harming yourself or the - Abused childhood
baby? Do you think life is worth living? Do you
hear/see things that others do not? Do you have any Management
strange experience? SADMA? - You have a condition called postpartum depression
- How are things at home? Do you have enough with some psychotic features. Our body and mind are
support from friends, family and husband? How is interconnected. When our mind is too stressed our
your relationship with husband? Any financial body starts showing symptoms and thats the reason
problems? why youre having tiredness, weight loss and sleep
changes. We did some investigations and all the tests
Diagnosis and Management are normal which means that there is no organic
- Most likely you have a condition called postpartum cause for your symptoms.
blues. It is more common during the first pregnancy - I have to admit you to the hospital. I will call the
and basically, it happens because of hormonal ambulance. In the hospital you will be reviewed by a
imbalance. There are also contributory social factors.
In your case, it is the lack of social support.
15

- psychiatrist. They will start you with lithium, anti- - Thought:


psychotic medications, and antidepressant o Stream: amount and speed of thinking
medications as well. If you dont get better with this o Form: how patient puts his thoughts across;
management, the specialist might also consider doing loose association/derailment, tangentiality,
ECT. flight of ideas, circumstantiality (over-
- I would also like to speak to your partner and other inclusiveness or beating around the bush)
family members because you need a lot of support. In content: delusion, thought
the hospitals, we have mother and child unit so you broadcasting/insertion/ideas of
can stay with your children. I would organize a social reference/suicidal ideation
worker to help you. Most likely, the breastfeeding will perception: hallucination and illusions
be terminated (bromocriptine dopamine agonist). - Cognition/Insight and Judgment
- Later on, you will also be seen by a psychologist and
they will consider doing CBT. Examination Findings
- According to the mental health act, I need to go for - My patient's general appearance is okay. She is
involuntary admission. I am sorry. casually dressed and well-groomed. Her behavior is
- Prognosis is good once treatment is administered but restless and agitated. Volume of speech is low and
there is a chance of relapse in the future. language is good. She describes her mood as "okay".
- Consider lithium prophylaxis in future pregnancies. Her affect is congruent with the mood. Her thought
- OFFER BABY CHECK! streaming is okay. She exhibits flight of ideas and has
delusions of grandeur. She believes that the president
Mania is going to die and she needs to save him. Perception
is alright. She does not have hallucinations or
Case: You are GP and a 35-year-old David comes to your clinic delusions. She is oriented to time, place and person
as his wife has some concerns about his behaviors. and has impaired insight.

Case 2: Your next patient is 24-year-old university student Differential Diagnosis


brought by parents who are concerned because of his change in - Mania
behavior for 2 weeks. He seems to have been hyperactive, - Drug-induced
sleeps less and has slept with 3 girls in the last 2 days. He has - Acute psychosis
been drinking a lot of alcohol and are worried he might get into - Hypomania
an accident.
Diagnosis and management
Task - Urine drug screen
a. History (6 minutes) - Admit patient
b. Explain to examiner differential diagnosis - Call psychiatric registrar: they might start him on mood
stabilizers
Differential Diagnosis
- Bipolar Mood Disorder ANXIETY DISORDERS
- Schizoaffective Disorder
- Personality disorder Generalized anxiety disorder (JM 1259/1260)
- Hyperthymic personality (no treatment unless they
come with mania or depression) Case:
- Alcohol/drug dependence - R/O drug dependence/withdrawal, hyperthyroidism,
- Anxiety disorder cardiac arrhythmias, pheochromocytoma, depression,
- Hyperthyroidism Diabetes

Treatment
- Admit - Do ECG, BSL and Urine at the office during his first
- Lithium carbonate, sodium valproate visit
- Anti-psychotic - DSM criteria:
- Anti-depressant (need to used with caution with mood o UNREALISTIC worries
stabilizer) o Uncontrollable worries
- Psychiatrist review and long-term followup o Symptoms are not the direct result of any
- Mental health care plan organic or psychiatric disturbances
o 3 or more symptoms:
Mania MMSE Irritability
Restless, keyed up or on edge
Case: Your next patient is a young uni student brought in by Easily fatigued
their concerned parents. She is insisting to fly to US to meet the Difficulty concentrating or mind
president. going blank
Muscle tension
Task Sleep disturbance
a. Mental state examination - Management:
b. Present findings to examiner o Relaxation techniques: YOGA AND
MEDITATION
MSE o Lifestyle modification: Diet
- Confidentiality o Physical activity: 30 minutes brisk walking
- General appearance and behavior: restless and most days of the week
agitated o Refer to psychologist for CBT
- Speech and language: rate, volume, quantity; fluency, o Sleep problems: sleep hygiene; may give
range of vocabulary short-term benzodiazepines (up to 2 weeks
- Mood and affect: congruent/incongruent; but usually 2 days to prevent drug
appropriate/inappropriate (related to situation) dependence)
16

o SSRIs after 3 months of lifestyle Breathing techniques to help


modification and CBT control panic attacks and
hyperventilation
Panic disorder with Agoraphobia Breathe in and out of paper bag
Acute attack: benzodiazepine
Case (Panic disorder): A 30-year-old female came in to your GP Lifestyle modification
attack saying she had a heart attack 4 months ago when she CBT: teach patients to
was visiting a postnatal class. All PE and investigations at that identify,evaluate and control and
time were normal. Patient does not understand her condition, modify their negative fearful
hence came to you for explanation. thoughts and behavior
SSRIs
Task - PANIC Disorder: Presence of both:
a. Focused history (cant breath, heart beating fast, o Believes having heart attack
fainted; anxious, stressed, other investigations normal; o Recurrent panic attack in which the onset is
given sleeping pills but did not take it; no longer going not associated with a situational trigger
out because she is scared of recurrence; worry about o At least 1 of the panic attacks has been
a lot of things says husband but she does not believe) followed by one month of one or more of the
b. No further examination following
c. Diagnosis Persistent concern about having
d. Management additional attacks
Worry about implications of the
Case 2 (Panic attack): Sheila aged 35 years presents to your attack or its consequences
GP clinic with history of sudden onset of palpitations, trembling, Significant change in behavior
sweating and chest tightness. She also had numbness and related to attacks
tingling feeling in her arms, legs, and around the lips. She had o Presence or absence of agoraphobia
similar episodes in the last few months and was investigated in o Panic attack are not due to the direct
the hospital with negative results. physiological effects of a substance,
medication, general medical condition or
Task another mental disorders
a. Focused history (2nd attack, was sitting in the staff
room and started feeling palpitations, trembling, Differential Diagnosis
sweating and chest tightness; lasted about 30 - Panic attack
minutes, blood tests; ECG normal; coffee 5-6 cups, - Anxiety disorder
father with HPN and angina; works as assistant - Conversion disorder
teacher; feels stressed but able to manage it;
relationship with partner is okay; no financial Agoraphobia: extreme anxiety about being in a place where
constraints; mood is okay; sleep and appetite is good; escape might be difficult or embarrassing. Extreme cases lead
life is still worth living; smokes 5-10 cigarettes/day; to a situation where the person rarely leaves home
drinks 2-3 SD/day)
b. Physical examination (looks anxious but well; PR 74, History
BP 110/70, thyroid normal; chest and heart normal; - Confidentiality
abdomen normal) - Please tell me more about what happened 4 months
c. Differential diagnosis and management ago? Where were you at the time? Can you describe
the environment? How many people were in that
Features (DSM IV) room? Was it a crowded place? Where was the baby
- Panic Attack Criteria at that time? What symptoms exactly did you feel?
o Discrete period of intense fear or discomfort Was that the first episode? What happened
in which four or more symptoms develop afterwards? Did you lose consciousness? I
abruptly and reach a peak within 10 minutes understand from the notes that you had a lot of tests
SOB or smothering sensations done and the results were all normal.
Dizziness, unsteady feelings, light - During the last 4 mos, have you had similar symptoms
headedness or faintness (e.g. palpitation, chest tightness, DOB, dizziness)?
Palpitations or accelerated heart Where? What situations? Environment?
rate - Can you describe yourself in terms of your
Trembling or shaking personality?
Sweating - Are you worried about having a similar episode again?
Feeling of choking - Did you attend more classes afterwards?
Nausea or abdominal distress - Do you think you are evading public spaces/closed
Depersonalization or derealization spaces since then?
Numbness or tingling sensations - How is your mood/appetite/sleep? Are you able to
Flushes or chills perform daily household activities?
Chest pain or discomfort - Have you thought about harming yourself or others?
Fear of dying - Hows the baby doing at the moment? Are you coping
Fear of going crazy or of doing well? Are you breastfeeding? Do you have enough
something uncontrolled support at home?
o Organic disorders that stimulate a panic - PMHx: thyroid, heart, mental illness; SADMA
attack are hyperthyroidism,
pheochromocytoma and hypoglycemia Physical Examination
o Management: - General appearance
Reassurance and explanation - Vital signs
Support - Thyroid examination
- Chest and heart
- Abdomen
17

Management - The psychiatrist may also consider giving SSRIs to


- Most likely you have a condition called panic disorder relieve your anxiety and maybe some short-term
with agoraphobia. It is a feeling of excessive anxiety benzodiazepines to help you with your sleep.
that causes symptoms like palpitations, sweating, etc. - Do not worry. The prognosis is good.
Usually, there is no cause for this problem. - Am I crazy? No. this is an anxiety disorder. Most likely
- I will refer you to a psychologist. We will try and help because you are too stressed at work. Take things
you to overcome this problem as it makes it difficult for lightly and do not bottle up things.
you to go out in public. We will teach you to how to - If you want to take time off, it is a very good decision.
avoid these attacks and what to do if you have one. - Relaxation and Meditation
This treatment is called CBT. - Review and reading materials
- We will also teach you some breathing techniques that
will help whenever you are stressed and anxious. Obsessive Compulsive Disorder
- If your symptoms are not relieved by CBT, the
specialist might start you with SSRIs. Also, if you Case: A university student came to see you at a GP clinic. He
develop recurrent attacks with increased severity, failed his exam recently.
SSRIs are required.
- I can ask a social worker to come and visit you at Task
home. a. Take a history
- Reading materials; Review materials and followup b. Diagnosis
with psychologist c. Management

Obsessive Compulsive Disorder (OCD) History


- I understand that you have come here because you
Case: a 30-year-old nurse came in to your GP clinic. She was worried about failing your exams. I am sorry about
seen by your colleague a week ago. She has repetitive, this. Why do you think it happened (I have trouble
frequently intrusive thoughts of washing her hands very studying because of my thoughts)? Can describe
repeatedly because she thinks her hands might be these thoughts for me? (I study history and whenever I
contaminated and transmits infection to patient. Investigations READ THE WORD KING, I feel anxious and I need to
were done and she was diagnosed with OCD. Shes feeling go around the room once). When did it happen? Since
anxious and frustrated and now thinks whether she can take when did you start having this condition? How often
some time off from work. does it happen? How do these thoughts make you feel
(They are unwanted thoughts. It distresses me and
A university student comes and when someone tells king, he makes me anxious.)? Have you tried to resist these
starts to walk. thoughts (Yes, I tried but failed)? Do you think that
these thoughts are unreasonable and excessive?
Task Apart from walking around the table do you have any
a. History (patient wearing gloves) other habits or rituals which you need to carry out
b. Management plan every day (He needs to wash hands 10 times a day).
Does it cause any trouble? Hows your mood? Hows
History your appetite? Do you enjoy the things you used to
- Confidentiality! like? Are you in a stable relationship? Are you
- Can you talk a bit more about it? Do you think it is sexually active?
rational to keep washing hands? How is it affecting - Risk assessment: Have you ever thought about
your life? Do you have any other repetitive thoughts? harming yourself or others? Do you see/hear things
Since when? Are you a perfectionist? when no one else is around? Do you think somebody
- Mood: I know this is a frustrating time for you but how try to put thoughts in your mind or steal from your
is your mood these days? Sleep? Appetite? Weight? mind? Have you excessively worried about simple
Anhedonia? things? Are you perfectionist?
- Have you ever thought of harming anybody or - Past History: Any medical condition? Particularly
yourself? Are there any times when your mood is thyroid. Any mental or behavioral problems in the
really high? past? SADMA? Who do you live with? Do you have
- Do you see/hear/feel things which others dont? any many friends? Hows your family? Apart from study do
strange experiences which others find hard to you have other interest? Are you financially secured?
believe? - Family History: Any medical or mental condition runs
- Insight: do you think you need help? in the family?
- Judgment: fire or envelope question
- Social history: how is the situation at work (stressed)? Diagnosis and Management
Home? Whom do you stay with? Financial issues? - Andrew, from my history I think most likely you have
Happy family? an anxiety disorder called obsessive compulsive
- SADMA disorder. Have you ever heard about it? It is a
common condition often starting in early adolescence.
Management People with this OCD experience recurrent and
- You have a problem called OCD. Do you know what it persistent thoughts, images or impulses that are
is? OCD is one of the anxiety disorders. It is a intrusive and unwanted. They also perform repetitive
common condition. People experience recurrent and and ritualistic actions that are excessive, time
persistent thoughts (obsession) leading to anxiety and consuming and distressing.
compulsion. - I would like to refer you to a specialist for assessment.
- I would like to refer you to a psychiatrist and Often the first step in treatment is psychotic treatment.
psychologist. They will do Behaviour Modification CBT (cognitive behavior therapy): The important part
Therapy (Exposure and Response Prevention) which of this therapy is gradual exposure to situations which
means the therapist will expose you to the obsession trigger obsession plus teaching behavior techniques to
and will teach you how to overcome your anxiety. reduce compulsion and anxiety.
18

- Medical treatment from psychiatrist: SSRIs o Psychological component: constantly


- Would you like me to organize family meeting? Do you worried, sleeplessness, restless, lack of
need any support for studying? concentration
- Prognosis: - Constellation of symptoms that persist for 1 month
o 20-30% will improve significantly. after exposure:
o 40-50% will have moderate improvement. o Acute PTSD: duration of symptoms <3
o 20-40%, they have chronic OCD or months
worsening symptoms. o Chronic PTSD: duration of symptoms >
- People who can have a good prognosis: 3months
o Good social environment o Delayed onset PTSD: onset at least 6
o Clear precipitating event months after stressor
o Episodic symptoms - Experienced or witness a traumatic event (death,
near-death experience, rape, earthquake, natural
Delayed-Onset PTSD calamities) with situation with feeling of helplessness
and extreme fear
Case: Paul is 55-years of age and a father of four. He presents - Typical distressing recurrent symptoms
with difficulties sleeping and concentration. He says that he o Recurrent intrusive features recollection,
wakes up in a cold sweat every night after a nightmare and he nightmares, flashbacks
finds it difficult to sleep because of anxiety. The nightmare is o Avoidance of events that symbolize or
always the same: a car accident like the one he experienced 10 resemble the trauma, detachment, feelings
years ago. Paul was driving from the hardware shop with his of numbness or withdrawal, guilt
then 8 year old son and suddenly hit on the side of the car and o Hyperarousal phenomena: exaggerated
swerved onto the side road. He remembers time slowly down startle response, irritability, anger, difficulty
as the car was sliding off the road, thinking this is it and turning with sleeping and concentrating,
to see his son leaning towards the window. He had wanted to hypervigilance
reach out and grab him but had to keep his hands on the wheel - Management
and regain control of the car. He said it was probably the most o Meditation and yoga
intense and scariest few seconds of his life. No one was injured o Lifestyle modification (smoking cessation,
and at the time Paul felt relieved, thinking for some time after decrease alcohol intake, healthier food, and
how lucky they were that the car stopped before crashing into exercise)
the nearby tree. Paul says he has not thought much about the o Proper sleep hygiene
accident until 2 months ago when his son came with his new Recognize what helps patient to
drivers license. He says he felt anxious and worried about his settle best, establish a routine
son driving, despite telling himself that his son is a safe driver. before going to bed, regular
Every time his son goes out with the car, he has some difficulty daytime exercise and time of
concentrating on anything else and is generally jumpy and on arising, avoid daytime naps, avoid
the edge. Then about a month ago the nightmares started. In strenuous exercise close to
the nightmare, he sees the accident, feels time slowing down bedtime; avoid alcohol and drinks
and sees his sons head smashed into the window. He wakes containing caffeine in evening,
up sweating, his heart palpitating, breathing heavily, and in avoid heavy evening meal and
complete panic. He describes the whole experience as so real smoking; remove pets from
that often in the morning he is overcome with a strong sense of bedroom; avoid lights including
grief. He is not sure whether it is lack of sleep or the dreams but poorly screened windows and
he has been walking around the home like a zombie and not highly illuminated clocks in the
really feeling quite there. He finds it difficult to feel positive room
about any activities and feels numb towards his wife. He Sleep promoting adjuvants: warm
describes the situation as unbearable. bath, warm milk, comfortable quiet
sleep setting with right
Task temperature, sex
a. Probable diagnosis Non-pharmacologic treatment:
b. Management advise (establish SAD, mood, suicidal, meditation, relaxation therapy,
stress management; CBT and
Differential Diagnosis electromyographic feedback;
- Acute stress disorder (constellation of abnormal hypnosis
anxiety-related symptoms occurring within a 4-week- o Crisis Intervention Therapy (CBT)
period. Symptoms are grouped as hyperarousal o Refer to psychiatrist and psychologist for
phenomena, avoidance of reminders and intrusive counseling (EMDR Eye Movement
phenomena; management: debriefing and counseling) Desensitization and Reprogramming)
- Adjustment disorder (anxiety symptoms within 3 o Social support
months of response to an identifiable psychosocial o Pharmacologic: SSRIs; short-term
stressor; persists for less than 6 months following benzodiazepines (sleep)
removal of stressor; non-pharmocological: counseling,
relaxation and stress management; pharmacological: Post-traumatic Stress Disorder 2
short-term benzodiazepines)
Case: Your next patient is a 22-year-old man who presents with
Diagnosis SOB and poor sleep. SOB occurred at night and relieved by
- Type of ANXIETY disorder waking up. He had a major vehicular accident 3 months ago and
o Physiological component: autonomic broke 3 ribs. He recovered well.
hyperactivity (palpitation, increased HR, dry Task
mouth, upset stomach) a. History (SOB last night, relieved by salbutamol, went
for a walk and felt better but returned in the morning
consult; stopped socializing and had no contact
with the GF)
19

b. Diagnosis and management Melbourne, recently divorced 3 weeks ago, not on any
contraceptives, regular with pap smear, no bleeding
History disorder; drinking alcohol 3-4 glasses of wine to help
- When did it start? Any previous episodes? Did it occur with sleep)
after the motor vehicle accident? Any triggering b. Physical examination (looks well without eye contact,
factor? Any associated features such as chest pain or VS normal, all PE normal)
sweating? How is your general medial health? Any c. Investigation
serious medical problems in the past? d. Diagnosis and Management
- I know you had a car accident, Im sorry about it. Im
not asking you to recall the event, but can you please Differential Diagnosis
tell a few words about it? Was somebody with you at - Major depression
that time? How is she? Do you have any contact with - Acute stress disorder
her? Do you experience any sudden images of the - PTSD
event? Do you try to avoid driving or the place where
the incident took place? Do you have any nightmares? Stressors:
Do you feel irritated, angry or guilty about anything - Death of a loved one
pertaining to the incident? - Divorce or problems with relationship
- How is your mood? Sleep? Appetite? Weight? - General life chages
Anhedonia? Suicidal ideation: do you think life is worth - Illness or other health issues in yourself or a loved one
living? Have you thought of harming yourself or - Moving to a different home or city
anybody else? Do you see/hear/feel things others - Unexpected catastrophes
cannot or have you had any strange experiences? - Worries about money
- Whom do you stay with? Do you have a lot of friends? - In teenagers: family problems or conflict, school
SADMA? problems, sexuality issues
- ENCOURAGE patient by nodding and showing
concern Features:
- 5 symptoms for 2 weeks daily: SAGECAPS (in
Diagnosis and Management depression)
- From the chat we had, I think you have a condition o Sleep
called post-traumatic stress disorder. Have you heard o Anhedonia
about it? It is a type of anxiety disorder where the o Guilt
patient experiences various symptoms and behaviors o Energy (lack of)
like recollection, flashbacks, avoidance, sleep o Concentration
problems following a psychologically distressing event o Appetite
which in your case is the MVA you had 3 months o Psychomotor retardation
before. o Suicidality
- The symptoms usually come immediately after the - Criteria
event but can be delayed for months or years. o Development of emotional or behavioral
- I will refer you to a psychiatrist. He will talk and listen symptoms in response to an identifiable
to you and will use some techniques to help you come stressor within 3 months of the onset of the
out of this situation cognitive behavioral therapy. stressor
As you havent slept for a few days, he may offer you o Symptoms or behaviors are clinically
sleeping pills for a short time but I would advise you to significant as evidenced by:
start with sleep hygiene. marked distress that is in excess
- At this stage, you might not need any medication, but of what would be expected from
if required, the specialist might offer SSRIs. exposure to the stressor
- I would like to do a family meeting if its okay with you. Significant impairment in social or
Family support is very important at this stage. occupational functioning
- Safe level of drinking. o Stress-related disturbance does not meet
- Review. Reading material about PTSD and sleep criteria for another specific axis I disorder
hygiene. and is not merely an exacerbation of pre-
- Differentials: anxiety disorder (GAD, adjustment existing axis I or II disorder
disorder, panic attacks, substance abuse) o Symptoms do not represent bereavement
o Once stressor has terminated, the
ADJUSTMENT DISORDERS symptoms do not persist for more than an
additional 6 months
Adjustment Disorder - Treatment
o CBT
Case: Your next patient in GP practice is 32-year-old Shirley o Relaxation technique (yoga and meditation)
Coombs complaining of SOB. She has recently moved from o Healthy diet and exercise, reduce caffeine
Sydney to Melbourne with prolonged travel time. o Stress management (dont bottle things up!)
o Sleep hygiene
Case 2: lady with chest tightness and pain and normal o Short-term course of drug treatment is
investigation husband died recently; necessary in persistent or severe case

Case 3: sad woman anniversary grief Developmental Disability with Adjustment Disorder

Task Case: You are a GP and your next patient is a 26-year-old with
a. Focused history (started 2 days ago, comes and go, Down Syndrome living in a support home. There is change in
does not change with position, started 4 weeks ago, behavior recently and he is very tired. He is afraid to coming to
present at rest, not feeling comfortable, no fever, feels the GP and that is the reason he hasnt come today as well.
tired, and breathless, drove from Sydney to Instead, there is a legal carer who has come to see you.
20

Task DSM IV criteria:


a. History from legal carer (he is not active, tired and - Eating amounts of food larger than most people eat in
doesnt want to get out of bed recently since last week a short period of time
when his close friend left the home and moved - Lack of control over eating
interstate; weight loss; no medications; mood is sad; - Eating is followed by compensatory purging by the
cant express in words because is mentally changed) following mechanisms:
b. Diagnosis and management o Self-induced vomiting
o Laxative abuse
History o Periods of fasting
- Confidentiality o Excessive exercise
- I understand that you are concerned that you noticed - Distorted self-evaluation of body shape and weight
changes in your patients behavior and that he is very
tired. Types of bulimia
- Could you please explain what you mean by change - Purging
in behavior? When did it start? Did anything happen - Non-purging: fast, exercise, abuse appetite
around that time when it started? How is Johns suppressant
mood? Does he have the capability to express - Commonly seen in people with depression. They
himself? What about his weight? Appetite? Sleep? usually have a history of neglect or sexual abuse as a
Has he ever thought of or done something to harm child or young adult. It is commonly seen in well
himself? educated high achieving females (compensating
- Do you think he has fever? Any chest pain or tummy mechanism). There is a coexisting history of alcohol
pain? Does he complain about anything? Any change abuse, social functioning impairment, along with FHx
in bowel motion, color of the stool, or waterworks? Is of mental disorders. 50% of these patients are also
there any smelly urine, frequency or urgency? Any diagnosed with a borderline personality disorder.
weather preference especially cold intolerance? Any
change in color of the skin? History
- Any history of getting heart problems? Has he traveled - Confidentiality
recently overseas or anywhere else? SADMA? Does - How is your appetite these days? Can you describe
he have family? Do they visit him? Does he have your typical daily diet to me? What is your perception
other friends in the support home? about your weight and your body image?
- Has there been a recent change in your body weight?
Management Do you ever lose control over how much you eat? Do
- Most likely, he has a condition called adjustment you force yourself to vomit? How frequent? Since
disorder. Because he was emotionally attached to his when have you been doing this? Have you ever used
friend, he might have develop these symptoms when any meds to lose weight (laxatives, water
he left but it could be depression resulting from pill/diuretics)?
hypothyroidism which is common in patients with - Any change to your period? LMP? Regular cycles?
Down syndrome. I would like to run some basic - How is your mood recently? Any changes? Do you
investigations for him including FBE, TFTs, BSL, iron sleep well? Are you able to perform daily life
studies, urine MCS, U&E, and LFTs. activities?
- At this stage, I want to see him. If he doesnt come, - Are you in a stable relationship? Any sexual
then I would like to arrange a home visit. If he has a problems?
family, I can arrange a family meeting because he - Do you feel your life is worth living? Have you ever
needs a lot of support right now. If you are really thought about harming yourself or others?
concerned, I can refer him to the counselor to help - Do you see/hear/feel things that others dont? Any
him deal with stress. strange experiences?
- I would encourage you to engage him in his favorite - Some people have a history of sexual abuse as a
activities. child. Do you think it may be related to you?
- I would like to review him once the results are back. - FHx: mental illness?
- Reading material. - SADMA?
- Red flags: for severe depression
Diagnosis:
EATING DISORDERS - You have condition called bulimia nervosa which is a
part of eating disorders. It is uncontrolled episodic
Bulimia Nervosa rapid ingestion/intake of large quantities of food in a
short period of time. The patient then feels guilty and
Case: You are a GP and your next patient is a 26-year-old afraid of gaining weight. They induce vomiting
female referred by a dentist because of poor dentition, dental themselves or might use medications/laxatives. It is
hygiene, dental carries and repeated vomiting. very common in young females.
- It is a risky condition. Repeated attempts to lose
Case 2: You are a GP and a 26-year-old computer analyst weight cause a fluctuation in body weight which
comes in complaining of self-induced vomiting and some affects all body systems especially the hormonal
changes in dorsum of hands. BMI is 24 balance. It can cause:
o Irregular periods
Task o Depressed mood
a. Relevant History (toothache, filling, self-induced o Loss of fluids and minerals
vomiting; dont want to get fat since teenage; mood: o Dental decay
sometimes low eat more, more depressed; normal - What we need to do is refer you to a psychiatrist. They
sleep will confirm the diagnosis first and then start treatment
b. Explain Diagnosis such as CBT and sometimes medication like SSRIs.
c. Management You need to come for regular follow up. We have
support groups for patients with eating disorders.
21

- Reading materials - Support group: Eating disorder foundation Australia


- Admission
o Affecting hormone system Eating Disorder
o Cardiac disease
o Suicide Case: You are a GP and the mother of a 13-year-old girl has
o Hormone changes come to you because her daughter has lost weight recently. She
does not feel hungry at the time of meal and her periods have
Bulimia Nervosa not come up to now.

Case: An 18 year-old-female has been admitted to the hospital Task


in the ED for Diabetic Ketoacidosis. Now she is ready to be a. History from mom (BMI 17, fear of being fat, excessive
discharged. She has normal weight and has a history of binge exercise, constant checking in the mirror, switched to
eating and self-induced vomiting and laxative abuse. vegetables)
b. Diagnosis and management
Task
a. Relevant history Case: Karen aged 16 years comes to your practice with her
b. Explain the condition to the patient and the mom Julie. Julie tells you that she is quite concerned about
management Karens eating habits. On your questioning to Karen, she tells
you that her mom isf forcing her to eat all the time and she is not
History happy and wants you to help her. Karen is a year 10 student in
- When were you diagnosed? Medication? Under local school and lives with her parents at home.
control? Last reading of your BSL? Is she on constant
follow up by her GP? Diagnosed two years ago. Task
Insulin injections. Missed some doses not regular. Do a. Further history (wants to lose weight because she
you check your sugar level everyday? How many thinks she is fat; does not socialize with friends; mood
times per day? Do you skip meals regularly? Have okay, no psychotic features)
you ever been admitted to the hospital with b. Physical examination (BMI 16.5, BP 100/70, postural
complication of DM? When was the last time you drop 15mmHg, HR: 56/minute; T: normal, oxygen
visited your GP (2years back. After Ive been saturation, BSL 4.6)
diagnosed I never went back)? Could you please tell c. Probable diagnosis and Treatment Advise
me about your daily meal for one whole day (She eats
2-3times a day and large amount of food). Do you feel Common Adolescent Problems
guilty about your eating habit? What do you do about - Asthma
it (She induces vomiting.)? How often do you vomit? - Obesity
Do you take any laxatives (Yes, 3 times per week)? - Eating disorder (0.3% for anorexia and 1% for bulimia)
What do you think about your body weight (Very fat I
cant look at myself in the mirror)? Early Warning Signs for Eating Disorders
- Periods: Are they regular? When was your LMP? - Concerned about food, dieting and exercise
Sexual history if she is sexually active? Do you use - Frequent weighing
any contraception? Do you think you could be - Refuse to join other family members in the table
pregnant? Psycho social history. Hows your general
health? SADMA? Differential Diagnosis
- Malabsorption
Counseling - Thyroid disorders
- Susan, you are suffering from two conditions: - Diabetes
Diabetes and eating disorder most possibly you have - Malignancy
bulimia nervosa. Have you heard about it?
- Its a common condition in your age group. A person Admission Criteria
with this disorder eats large amount of food in a short - Electrolyte imbalance
time the loose control and cant stop eating. After that - Suicidal ideations
they induce vomiting to compensate. Its a very risky - Severe dehydration
situation for you as you are not taking your meals and - Hypothermia (<35C)
insulin regularly. You had a complication of diabetes - ECG showing arrhythmias
called DKA for which you were admitted to the ED. - Postural hypotension (>10)
When you induce vomiting you only loose water and - Bradycardia (<50/minute)
salts from your body but not body fat. Its dangerous
as it can cause damage to your kidney and cause History
water and mineral imbalance. In the long run, it can - When exactly did you start to realize that her weight is
affect your heart, lungs, eyes and nerves also. Eating less than expected? Do you know her weight at the
disorder can affect your health adversely, it can cause moment? Can you describe her typical daily diet? Is
dental problems, I know your main concern is your she eating vegetables and fruits only? Is there any
body weight. Let me assure you we will help you to intake of meat as well? Can you describe what you
achieve an ideal body weight without having to induce mean when you say that shes not hungry? Does she
vomiting or use laxatives. We will work as a eat fast food? Did you try to invite her to eat with the
multidisciplinary team. I will refer you to a dietician family? How is her school performance? Is she still
with whom you can discuss an appropriate diet plan engaging in sports and other activities? Does she
for you. I will also refer you to endocrinologist to adjust exercise? How much? How many days a week? What
your insulin dosage. type of exercise?
- Psychologist for CBT. With your consent I can arrange - How is the situation at home? Any stress like financial
a family meeting as family support will be essential. Id or emotional problems within the family. How is her
also write a letter to your GP so that he can follow you relationship with you and with the rest of the family?
up regularly and we can prevent complications in the Does she go out with friends? Does she enjoy their
future. company? Do you know if she is sexually active?
22

- Please tell me more about her period problem? Any Task:


spotting/bleeding? Any lower tummy pain? Have you a. Focused history
noticed that she has become mature physically b. Investigate/examination (jug, glass, pen torch, tongue
(breast, armpit and pubic hair development)? What depressor)
was your age when you had your periods? c. Diagnosis
- Can you describe her mood to me? Have you ever d. Management
noticed that she cries or irritable at times? Sleeping
problems? Has she told you that shes able to see or DIFFERENTIAL DIAGNOSIS for aphonia
hear things that you dont. a. Hoarseness
- Can you feel how she feels about her body? Do you b. Lump (thyroid)
think she is scared of becoming fat? Does she deny it c. Vocal cord paralysis
when you try to tell her shes becoming thin? Has she d. Psychiatric
ever talked about harming herself? Have you noticed e. Tumor
that she eats large quantities of food and vomits f. URTI
forcefully? Does she have access to any kind of g. Overusage of voice
medications especially laxatives or water pills? Has
she ever complained of dizziness or loss of Focused History
consciousness? Any other past medical or surgical - Start with confidentiality statement (station 7, 19)
condition? Any problems with waterworks and bowel everything we talk today would remain between you
habits? Are you aware if she smokes or drinks and me. I will only breach the confidentiality if you are
alcohol? Do you think she might be using recreational in danger, if somebody else is in danger because of
drugs? Is there a family history of eating disorder or you, and if I have to give this statement in the court
psychological problems? - I understand that you are worried about your moms
condition and I am sorry to hear about her
Criteria: - I understand that you lost your voice while singing
- Refusal to maintain normal body weight at or above a prayers for your mom
minimum normal weight for age and height (<85% of - I would like to ask you some questions. I would
expected BW) appreciate it if you could nod your head or write it on
- Intense fear of gaining weight or becoming fat despite paper.
current underweight status - Any problems like this before? Can you make a sound
- Disturbance of body image (body size or shape) (look for hoarseness)? Can you cough (nerve
- Amenorrhea in postmenarchal females (absence of at paralysis)? Singing or shouting aloud? Runny nose or
least 3 consecutive cycles) flu-like symptoms? Feeling of lump in throat making it
- Types: restricting type OR purging/binge eating type difficult to talk?
- Mood questions: is your mood low? Any sleep
Management problems? Have you lost or gained weight? Changes
- From the history, most likely your daughter is suffering in appetite? Suicidal ideations (do you think life is
from an eating disorder. It can be of many types, most worth living? Have you thought of harming yourself or
probably, her eating disorder shows some anorexic anybody ever?)
features. It is important to diagnose any eating - Psychotic symptoms: do you hear or see things which
disorder as early as possible because the outcome other people do not? Unusual experiences?
improves. If possible, bring her to me on your next - SADMA
visit. If you like, you can see a psychologist straight - Take informed consent for physical examination
away. I can write a referral letter for you.
- The treatment involves CBT (FBT more effective) Physical examination:
sometimes with the use of medications such as - Sip water and swallow when you ask her to swallow
SSRIs. I will also write a letter to a dietitian. She will - Push out the tongue to look for tongue deviation; gag
make a diet chart for her. With the help of a diet chart, reflex; uvula; throat
she can eat a healthy balanced diet without putting on - Do other cranial nerve examination; ENT;
weight. chest/heart/abdomen
- Please remember that she will need family support
throughout the way because sometimes, eating Diagnosis/Management
disorders may be complicated by depression, - From history and examination, the sudden loss of
electrolyte disturbances, nutritional deficiencies, as voice in your case is because of a condition called
well as drug and alcohol abuse. conversion disorder (aphonia).
- We will work in a MDT approach with myself as her - This results from an unresolved emotional conflict
GP, counselor/psychologist and dietitian in order to where thoughts or experiences are unacceptable to
help her. the mind resulting into a physical symptom.
- Investigations; FBE, TFTs, Dexa Scan, Hormone - Draw the body-mind axis body and mind are inter-
study, U&E (calcium, magnesium, potassium, connected. The stress from concern of your moms
phosphate, sodium), ECG condition has resulted in bodily symptoms. Whatever
youre experiencing is real but we could not find an
SOMATIZATION DISORDERS organic cause for it. For example, students start
getting tummy pain or vomiting when exams come up.
Conversion Aphonia - Dont worry. This has a good prognosis. We can treat
and cure it. I will refer you to the psychiatrist and
Case: A young 18/F presents with sudden loss of voice. Her counselor who will help to relieve your stress and
mother was diagnosed with end-stage liver cancer with anxiety.
metastases. She lost her voice when she was singing prayers in - If you are happy, I can organize a family meeting to
church for her mother. discuss your situation with the family and if the family
agrees, I will contact the oncologist who is looking
after your mom.
23

- Consider ENT referral to rule out organic pathology - Dont worry. You made a decision to come up and talk
- Review after couple of weeks about it. At this stage, I would like to refer you to a
psychiatrist who will do talk-therapy. He may also put
Critical Errors you on some medications (SSRI and antipsychotics)
- Not sympathetic for your anxiety and concern.
- Did not rule out organic causes - I would advise you to remove the mirrors in your
- Does not know the diagnosis home.
- Offer family meeting/support/social workers
Body Dysmorphic Disorder - Review, reading material, referral.

Case: You are an HMO and your next patient is a 29-year-old Case 2:
male who came to see you. He brought an envelope containing
hair and he asked you to examine it under the microscope. He is History
worried that he is getting bald and he believes that this hair loss - Confidentiality
will affect his promotion. - History: patient information HPI
- Family history
Task - Personal and social history
a. Focused history - MSE
b. Diagnosis o Appearance: eye contact; psychomotor
c. Management agitation, how patient dresses,
o Speech: pressured, normal, soft, loud,
Case 2: Female wants to have a breast reduction surgery monotonous
o Affect
Case 3: Man concerned about penis size o Thought: Form and Content
o Cognition
Case 4: You have a 24-year-old male student coming to your o Suicidal Risk
GP clinic asking for referral to plastic surgeon because he thinks o Insight/Judgment
his nose is too big. On examination, you find that his nose is
completely normal. Management
- I think you have what we call BDD. This means that
Task you have a preoccupation with a certain part of your
a. Further focused history body even though it is normal and this is causing you
b. Examination not necessary distress and anxiety.
c. Advise patient - I can give you a referral to a plastic surgeon if you
want, for a second opinion, but honestly at this point, I
Features dont find it necessary.
- Preoccupied - I would, however, like to refer you to a psychologist for
- Try to HIDE it cognitive behavioral therapy so that he can talk things
- Frequent mirror checking through and identify any stressors and help you cope
with them.
History
- I can see from the notes that you have hair in this Somatization Disorder with Agoraphobia
envelope and you want me to take a look at it under
the microscope? Why? Is it the first time? How is it Case: You are seeing a 26 year old female, Nardia, who comes
affecting your life? Apart from hair loss, are you to you for review of the result of MRI brain which was requested
concerned about any other part of your body? Does by the neurologist specialist. Nardia did the MRI as she has
this concern preoccupy you? Do you try to hide it? headache associated with severe neck spasm. Nardia has past
- How is your sleep? Restless? How is your mood? history of abdominal pain and nausea. Investigation with blood
Appetite? Weight? Anhedonia? Guilt? Do you think life tests, CT and U/S were all normal. Colonoscopy and upper GI
is worth living? Have you thought of harming yourself endoscopy were normal too. Past history revealed she had a
or others? Do you have any strange experiences? feeling that she had a breast lump on investigation that was
- Social history: how are things at home and at work? normal as well, history of chest pain investigation ECG stress
Do you have a lot of friends? Do you socialize? test echo and Holter were all normal. Nardias MRI results are
- SADMA? all normal

Examination: HAIR Task


a. Talk to patient regarding her complaint (Mood is flat.
Diagnosis and Management: Dad and mom left job and looking after her. She is
Case 1: eating well and putting on weight. Wont hurt herself.
- Your hair looks normal and let me reassure that I do Stays at home all day not interested to go out, no
not see any hair loss. It is normal to lose up to 100 friends or siblings. No hallucination or thought
strands of hair a day which is the natural process of insertion. Anxious and worried. Married before and
hair regrowth. I also understand that it is normal to be now divorced. Period regular, not sexually active (not
concerned about ones appearance, but in your case, interested) Been like this for 6-7 years. No smoking
you are over-concerned which is making you anxious alcohol and drug.)
and affecting your function. This is known as body b. Diagnosis and counsel accordingly
dysmorphic disorder which is a type of somatoform
disorder. In your case, it is accompanied by anxiety as Features:
well. Unfortunately, such an irrational belief can lead - History of symptoms for which significant medical
to severe self-doubt and low self-esteem. Your treatments have been sought
problem has been worsened by your work stress. - History of doctor shopping
24

- Appear before the age of 30 - I will give you some reading material about this
- Criteria: 4 pain symptoms, 2 GIT, 1 sexual, 1 disorder medically named somatization disorder with
pseudoneurologic symptom agoraphobia.
- Please understand that this treatment will take some
History time to work. Meanwhile, you can take some OTC
- May I ask more about the pain? How bad is it? Does it painkillers to relieve the pain.
go anywhere? Has it changed over the past few
hours? Is it the first time for you to have neck pain? Hypochondriasis
- I understand from the notes that you have had
previous treatments for different symptoms like Case: Suzanne aged 45 years presents to your surgery in a
headache, tummy pain, etc. any complaints now? busy afternoon. She tells you that she had right sided abdominal
What treatments were given? Did it help? At the pain for a few years and she bad been investigated in detail with
moment, how is your mood? Are you enjoying daily negative results. She describes pain is not present all the time
life activities? Are you socializing with family or and she thinks she has colon cancer and maybe doctors are
friends? Do you work? Are you able to concentrate unable to diagnose. On further questioning she denies weight
well on your work? Any difficulties in making loss, change in bowel habits or any history of melena or
decisions? How is your appetite these days? Sleep? hematemesis. Her appetite is good and is well otherwise. She
Weight? Are you in a stable relationship at the requests you to do another ultrasound and colonoscopy.
moment? Sexually active? Are you still interested in Suzanne lives independently in a flat and had no known medical
sex? or any surgical problems.
- I need to ask you some strange questions. Do you
see/hear things that others dont? Do you think that Task
someone is trying to harm you at any way? Have you a. How will you manage Suzannes request
ever thought about harming yourself or ending your
life? Have you thought of a plan? Have you bought Features
something to carry out that plan? Do you feel
excessively anxious about things? Do you feel SOB, Somatoform disorder
dizzy, or fainting at any time? Are you happy with your - DSM IV Criteria
life? Do you feel guilty about anything? Any bad o Preoccupation with fears of having or the
memories for you? Any childhood incidents that you idea that one has a serious disease based
cant forget? Any repetitive thoughts? Any other on the persons misinterpretation of bodily
medical or surgical conditions that I should be aware symptoms
of? Have you ever been diagnosed with depression or o Preoccupation persists despite appropriate
other psychological illnesses? FHx of psychiatric medical evaluation and reassurance
illnesses? SADMA? How do you support yourself? o Belief (A) is not of delusional intensity and is
Financial issues for you? not restricted to a circumscribed concern
about appearance
Diagnosis and Management o Preoccupation causes clinically significant
- I understand that you have pain in the neck along with distress or impairment in social,
stiffness. On the other hand, it seems like you are occupational, or other important areas of
unhappy and depressed. Im wondering if we can link functioning
the two conditions. In my opinion, the mind and body o Duration of disturbance is at least 6 months
are connected deeply. Whenever there is stress upon o Not better accounted for by GAD, OCD,
the mind, the body reacts by producing symptoms like Panic disorder, major depressive episode,
nausea, vomiting diarrhea before exams. Just like separation anxiety or another somatoform
that, you are having pain in your neck. This pain is disorder
quite real; however, the CT scan shows that there is
nothing physically wrong with the structures in the History
neck. - r/o organic disease
- We have a management plan which involves a - risk assessment: mood, and suicidal ideation
multidisciplinary approach. First, lets deal with your
pain. We can refer you to a pain management clinic Counseling and Management:
and give you a stronger medication. o Group psychotherapy and CBT
- Sometimes, talking about the stress and emotional o Lifestyle modification
conflicts within the mind can help to relieve these o Relaxation techniques
symptoms. So I would recommend for you to see a o SSRIs
psychologist for a type of treatment we call as
cognitive behavioral therapy/talk therapy. They might PERSONALITY DISORDERS
give you some medications like SSRIs or certain
anxiolytics. Antisocial Personality Disorder
- This problem in your neck can go away. I will see you
in two weeks to see how you are going. There are a Case: You are an HMO in ED when a police brings a 27-year-
few things you can do to reduce your anxiety: old Michael from a boarding house where he had been in a fight
breathing slowly, hyperventilation will make symptoms with another resident which he seems to have started and he
of panic attack worse, use relaxation technique, dislocated her middle finger. The police want him to be
exercise and swimming. Make a change in your life medically checked before they take him to the police station to
style. Reduce caffeine intake and alcohol. Exercise charge. Michael is well known to your hospital.
regularly.
- Refer to Anxiety recovery center Victoria or Metal
health organization of Australia
25

Task - Affect: congruent/incongruent;


a. Focused history (bored by what the resident was appropriate/inappropriate
watching on TV, he deserved to get punched - Thought form: flight of ideas, loose association,
switched the channel and punched him, mood is okay, derailment, tangentiality, circumtiality
frustrated, recently got out of prison) - Thought content: delusion, suicidal/homicidal
b. Review the hospital record (involved in a lot of fights ideations
at the age of 13, shoplifting, no medical illnesses, - Perception: hallucination and illusions
substance abuse, - Cognition, insight and judgment
c. Diagnosis and Management to examiner
Present to Examiner
Features - We have a 25-year old male who is well-dressed and
- Behavioral problem associated with a lot of arrest, wears a fluorescent green shirt. His behavior is
physical assaults, inability to keep a job, impulsive flirtatious. On examination, his speech is quality, rate
behaviors (gambling, promiscuity, alcohol, drugs) and volume of speech is within normal. He describes
- Impulsive, insensitive, selfish, callous, superficial his mood as sad and it is congruent with his mood. He
charm, lack of guilt, low frustration level, doesnt learn does not exhibit any abnormalities in thought form or
from experience, relationship problems, reckless, content. He denies any suicidal or homicidal ideations.
disregard for safety of self and other His insight is intact with regard to his problem but not
his personality. My provisional diagnosis is histrionic
History personality disorder. I would like to refer him to
- Confidentiality psychologist for CBT, counseling and support.
- What actually happened? How did you break your
finger? Did you feel bad about punching him? Do you Differential diagnoses
pick up fights easily? Have you been imprisoned many - Histrionic
times? Do you have problems with the law? SADMA? - Mania: extravagant, judgment impaired, insomnia;
Have you ever been cruel to animals before? - Drug-induced
- How is your mood? Suicide intentions? Psychotic
symptoms? MEDICATION, SIDE EFFECTS AND PROCEDURES
- Whom do you live with at home? Are there any
problems? Do you work? Any financial problems? Side Effects of Antipsychotics

Hospital record from examiner - Dopamine receptors - Amenorrhea Galactorrhea


- Reason for admission Decreased libido, gynecomastia (Increased prolactin
- Has he been referred to any psychiatrist (he doesnt - Histamine - sedation and weight gain
followup because he think hes not crazy) - Alpha Blockers - orthostatic hypotension
- Was he given any treatment? - Muscarinic Blockers - Atropine effect
- Diabetogenic - Olanzapine and clozapine
Differential Diagnosis and Management - EPSE - Dopamine goes down Ach goes up
- Substance abuse o Acute dystonia- within days Tx: Benztropine
- Antisocial personality disorder (anticholinergic)
- Intermittent explosive disorder o Parkinsonism
o Akathisia - generalized restlessness
Management (differentiate anxiety)
- It can be very difficult to manage. Aim is to manage treatment: BZDs or Beta Blockers
impulsive behavior and lower dose of antipsychotic
- Group therapy is important o Tardive Dyskinesia : chronic irreversible
- Drugs: SSRIs, mood stabilizers, and beta-blockers involuntary movements; disappear when
- As they grow older (>40), the condition gets better. patient sleeps
- 1st Generation have more EPS because they act
Histrionic Personality exclusively on Dopamine receptors but can also
happen for 2nd generation drugs.
Case: Your next patient in GP practice is a 25-year-old male - Clozapine : only antipsychotic drug that does not have
who tells you that his life is falling apart because his GF has let EPS and Endocrine SE's because it is not using
him. He cant sleep and hes thinking of her all the time. Hes substantia nigra
missing her and requests for help. Patient is wearing fluorescent o kept as last resort because of
green shirt, well-dressed. Agranulocytosis
o Indications for use: 2 drug failure; if patient
Task develops TD
a. MSE - Crossover for Antipsychotic drugs
b. Present to examiner - Washout period for Antidepressants to prevent
c. Management serotonin syndrome

Features: Serotonin Syndrome


- Seductive and flirtatious
- Attention-seekers Case: Your next patient in GP practice is a 27-year-old male
who is being treated for depression. He has been given a trial of
MSE medication without much improvement. He was started with
- Appearance: dress and grooming (makeup), flirtatious, Sertraline (Zoloft) 100mg OD. He is undergoing CBT with
anxious, restless, psychomotor retardation psychologist. His mood is the same with no improvement. He
- Speech: quality, volume, rate comes complaining of inability to sleep, nausea, and diarrhea.
- Mood: use patients words
26

Task - Moderate to severe: stop the drug, refer for admission;


a. History (changed medication to Zoloft from fluoxetine cold IV infusion and blanket for fever. For agitation,
because patient is not responding) benzodiazepines; for rigidity (dantrolene);
b. Examination (temperature 37.8, and pulse 90, BP methysergide or cyproheptadine. Once stabilized, we
120/80, RR normal, BMI normal, sweaty and tremors) can start the anti-depressants again.
c. Diagnosis and management
Tremor (Side effect of risperidone Parkinsonism)
Features:
- Increased serotonin due to overstimulation of Case: A 32-year-old woman comes to your GP practice. She
serotonin receptors has history of depression with psychotic features. She is on
- Most common cause is interaction between SSRIs risperidone and mirtazapine. She developed tremor three weeks
and MAO or inadequate washout period ago and she comes today for prescription renewal.
- Related to combining antidepressants and other drugs
such as tramadol, ecstasy, tryptophan Task
- Others: carbamazepine, lithium, tramadol, St. Johns a. History
wort, SSRIs, TCAs, venlafaxine, mirtazapine, b. Physical examination
amantadine, bromocriptine, cabergoline, levodopa, c. Explain condition and Management
pergolide, selegiline, cocaine, MDMA, LSD,
dihydroergotamine, sumatriptan, bupoprion, pethidine, History
reserpine - I understand you have come to see me for
- Clinical features: prescription renewal. Can you please tell me about
o Rapid onset of tachycardia, dilated pupil, your tremor? Which parts of your body does it affect?
increased reflexes, tremors, fever, sweating, Does anything make it better or worse? How does it
shivering, diarrhea, hypervigilance, affect your life? Does it interfere with your writing?
agitation, irritability, hypertension, confusion, Have you noticed any changes in the way you walk?
and convulsion, coma Do you have muscle stiffness? Do you have trouble
o Diagnosis is usually clinical initiating movements? How long have you been on
- Treatment: urgent referral these medications? Any recent changes in the dose?
o Meds: methysergide and cyproheptadine - Hows your appetite? Did you put on weight recently?
o Mirtazapine TCA quicker result Do you have dry mouth, constipation, or trouble with
o Trazodone causes sedation insomnia urination? Have you noticed milky discharge on your
Do not change frequently due to breast and menstrual irregularities? Do you feel
increased risk of resistant sleepy? How is your mood? Sleep? How is your social
depression life? Do you think life is worth living? Suicidal
ideation? SAD?
History - Who do you live with at home? Do you have enough
- I understand from the notes that you were diagnosed support?
with depression and given medications.
- Confidentiality statement. Physical examination
- How long have you been having these recent - General appearance: lack of facial expression,
symptoms? Did it start suddenly or gradually? When absence of blinking, flexed/stooped posture, drooling
were you diagnosed with depression? Were you on of saliva
any medication before this? Do you know the reason - Vital signs: especially BP (Postural drop)
why it was changed? How many weeks did you wait - Assess gait: look at the way they stand (might have
before starting the medication? Are you regularly trouble rising from the chair), shuffling/festinating gait,
taking the medications? Have you tried to self manage lack of arm swing, ask patient to stop, turn, and walk
or double the medication or change doses than what back
was prescribed. Are you taking any other medications, - Resting tremor (asymmetric; comment where
herbal teas or other OTC drugs? Have you ever tried dominant); ask for some mental test to check for the
using illicit drugs recently? Other symptoms? tremor; finger-nose test (intention tremor; resting
Waterworks? Bowel? tremor improved); fine tremor (stretch arm in front);
- How is your mood now? Do you think that life is still - Tone
worth living? Have you thought of harming yourself or - Bradykinesia test: finger tapping test (playing piano);
anyone else? Are there any times when your mood twiddling test; ask patient to write name (check
is very high? Any strange experiences? micrographia); glabellar tap (normal blink <3; people
- Whom do you live with? Are you living alone? with Parkinson will blink always)
- Speech: tone and volume (monotonous and volume is
Physical examination low)
- General appearance; agitated and may appear sick - Check for weakness of upward gaze; eyebrows for
- Vital signs sweatiness and seborrhea
- Eye: pupil eye and shape (mydriasis), PEARL
- CNS: gait, tremors, hyper-reflexia Diagnosis and Management
- According to history and PE, you have a condition
Investigations: FBE, UEC, LFTs, TFTs, urine and drug screen called parkinsonism. It is an adverse effect of
(with consent) risperidone. It is more common in older generation
antipsychotic. Typically, it develops during the early
Management weeks of treatment. However, it could be seen in
- This is most likely serotonin syndrome which is due to newer generation antipsychotics as well. I will liaise
inadequate washout of the drugs leading to excessive with your psychiatrist. Ideally, risperidone drug should
effect of the serotonin in your brain. be reduced and switched to a different antipsychotic.
- Mild: stop medication and review in 24 hours and
reintroduce sertraline at a lower dose
27

To relieve your symptoms, I can prescribe anti- akithesia (restless: motor


cholinergic medications, benztropine. However, side restlessness that occurs within
effects include blurred vision, dry mouth, constipation, hours of starting treatment; may
and urinary retention. subside spontaneously; tx: dose
- It takes one to two weeks. Your current medication reduction)
dose will be reduced while a new medication is parksinsonism (tremors, rigidity,
introduced and gradually increased until dose is bradykniesia); develops within
therapeutic. weeks to months; reversible; tx:
- Can it be done as outpatient? It could be done if you short-term use of benztropine;
have a good plan from psychiatrist, family meeting tardive dyskinesia: sudden
and involvement. The family should know the involuntary movements of the
symptoms of relapse and we would organize face, mouth and tongue;
community psychiatric nurses. irreversible; seen in chronic
schizophrenia who are on long-
Side Effects of Antipsychotics (Olanzapine/Fluphenazine) term tx; can come even after
stopping the medication; tx: switch
Case: You are a GP and a 23-year-old female came to you for to clozapine
consultation. She has been diagnosed with chronic neuroleptic malignant syndrome:
schizophrenia for some time. She has been on some meds that fever, muscle rigidity, autonomic
has been changed recently. She complains of weight gain. instability, more commonly seen in
Current BMI is 30. young males; tx: stop medications;
supportive treatment; ICU;
Task respiratory support; volume
a. History replacement; dantrolene
b. Diagnosis Fever >40
c. Management Encephalopathy
Vitals unstable
Case 2: Kelly aged 25 years presents to your surgery with Elevated enzymes
concerns about her recent weight gain. She had chronic Rigidity
schizophrenia which is managed by her psychiatrist. She had o Sedation: both typical and atypical; usually
been on fluphenazine for few years but recently was changed to improves with time; take medications close
olanzapine due to some tremors in her arms. Since the change to bedtime; avoid driving and operating
in medication she had no relapse of schizophrenia and even heavy machineries
tremor had settled. She is compliant with her meds. Kelly lives o Postural hypotension: more common with
with her mom who has been very helpful to her during her risperidone and chlorpromazine; more
period of illness. She lost her job a few months ago due to mild commonly seen in elderly people; tx: adjust
relapse of schizophrenia and has been unable to find a new job. dosage
She receives disability support pension. She is a smoker since o Weight gain: olanzapine/clozapine; monitor
age 17 but drinks alcohol on social occasions. Kelly had begun food intake; lifestyle changes; change
a new relationship but is concerned about her weight gain as medication if still not managed
her boyfriend seems unhappy and she fears losing her o Anti-cholinergic side effects: dry mouth,
boyfriend. blurred vision, headache, constipation,
Task urinary retention (improves with time);
a. Further focused history (shifted to fluphenazine due to o Endocrine side effects: hyperprolactinemia,
previous history of tremors with other drug) galactorrhea, diabetogenic
b. Physical examination (BMI 37, no thyroid o Hematological: agranulocytosis (clozapine);
enlargement) stop and change medication
c. Diagnosis and management advise o Sexual: retrograde ejaculation and
decreased libido; tx: does not improve with
Differential Diagnosis time; change medication
- Drug-induced
- DM History:
- Thyroid problems (hypothyroidism) - Schizophrenia: past and current history
- r/o PCOS in females - Medications: previous and current medications
(olanzapine)
Side effects of anti-psychotics - Other side effects: EPS, sedation, etc
- Atypical: olanzapine (zyprexa), clozapine (clopine), - General medical history
risperidone (risperdal), quetiapine (seroquel) - PMHx, sexual history, SADMA
- Typical: - HPI: can you tell me more about your problem? How
o Phenothiazides: chlorpromazine (largactil) many kg have you gained over how many weeks?
o Butyrophenones: haloperidol (serenace) Can you describe your typical daily diet? Do you do
o Diphenylpiperidines (pimozide) any exercise at all? How much? How many days a
o Thioxanthines: flupenthixol (fluanzol) week? What is your occupation? I understand from
o Trifluperazine (stelazine) the notes that you have been diagnosed with
- MOA: block dopaminergic neurons with additional schizophrenia. Since how many years? Who
effects of histamine, serotonin and acetylcholine diagnosed it for you? What symptoms do you have?
- Side effects: What medications were you started on? Dose? How
o Extrapyramidal long did you take those medications for? Any side
dystonia (spasms: trismus, effects? Why was it changed? I understand you were
torticollis, oculogyric crisis recently changed to olanzapine. What dose were you
within1st 24-48 hours of starting
medication), tx: benztropine
28

taking? Was it regular? Did you have any other side - Can you describe to me what exactly do you mean by
effects from this medication (abnormal movement dizziness? In what position do you feel dizzy? Did you
around the mouth or the body?noticed any shaking of lose consciousness at any time? Did you fall down
your hands? Restless? Sleepy more? Dizziness and hurt yourself?
especially on getting up in the morning? Any - I understand you went to your GP recently. What
complaints of dry mouth, headache, blurred vision? symptoms did you have at the time? Do you think you
How are your periods? Are they regular? LMP? can see or hear things that others dont? Do you think
- May I ask how is your mood? Sleep? Appetite? the TV or radio is talking to you or about you? How is
Energy level? Do you feel interested in daily life your mood these days? Any ideas about harming
activities? Have you ever thought of harming yourself yourself?
or others? I need to ask you some questions that - What medication was prescribed to you and in what
might sound strange or funny. Do you ever dose? Were you able to follow the instructions
see/hear/feel things or voices that other cant? Do you properly? Who looks after your medications for you
think the TV/radio are talking to you or about you? Do (wife)? What dose did you take? Do you have the
you think someone is spying on you? Do you think you prescription with you? Or do you have the bottle?
have special powers? Any past medical or surgical Which medication were you on previously? What was
history that I should be aware of? SADMA? Have you the name? dose? Any side effects? At the moment do
ever had your blood sugar checked? Any family you have any complaints of N/V/blurred vision/dry
history of psychiatric illness. Are you on a mouth? Any abnormal movements around the mouth?
relationship? What methods of contraceptive do you Do you feel restless all the time? Have you noticed
use? What contraception do you use? Any chance any tremors or shakes of your hands? When was your
you might be pregnant at the moment? last visit to the specialist psychiatrist? Any past
medical or surgical history? Any hospital admissions
Management previously? Have you ever suffered from high or low
- I would like to organize some blood tests on the BP before? SADMA (medication interaction: antifungal
patient. FBE with PBS, BSL, TFTs, LFTs, lipid profile, and SSRIs inhibit liver breakdown of risperidone
serum prolactin, U/E/C, ECG, CXR increase blood levels)?
- Most probably what you have is a side effect of - Am I able to talk to the carer or the wife?
olanzapine (zyprexa). All medicines have some side
effects but they are important to control your Physical examination
symptoms of schizophrenia. At the moment, I need to - General appearance: LOC (alert, confused, drowsy),
talk and liaise with the psychiatrist. They might decide pallor, jaundice, dehydration; any visible abnormal
to change this medication but before that, I want you movement of the face or the body? Tremors?
to try some dietary and lifestyle changes. Please try - Vital signs
regular exercise 30 minutes a day for most of the - Neck for LAD
week, check your weight regularly, and choose a - Chest/heart/abdomen
healthy diet. The dietitian can help by making an - BSL and Dipstick
appropriate diet chart for you. This weight gain puts
you at a high risk to develop DM, HPN, and heart Management
disease, depression (obesity can cause non- - You have a condition called postural hypotension.
compliance of medications) This condition is characterized by a change of your
- You need to have regular followup with me and the blood pressure while standing up from a sitting
psychiatrist. If they decide to change the medication, position that gives you dizziness. There are two
you will need to be admitted to the hospital during the possibilities: either you are having a side effect of
crossover period which usually takes around 2 weeks risperidone or a higher than normal dose has been
(tapering previous medication slowly and introduce the taken. The side effect is quite commonly seen within
new one while observing for side effects). the first week of treatment even at a normal dose
- Diet and lifestyle should be done until she has seen (usual starting dose: 1mg BD gradually increasing to
psychiatrist. 3mg BD).
- Is it the doctor who has done some mistake? Your
Postural Hypotension (Side Effect of Risperidone) doctor has prescribed what is best for you. I would
prefer to take a look at the prescription and if possible,
Case: You are an HMO and a 25-year-old male came in to the I will talk with your wife who takes care of your
ED due to dizziness since this morning. He has been diagnosed medication.
with schizophrenia for the last 10 years and his symptoms are - What we need to do now is to keep you in the ED to
usually controlled with medications. Recently, his wife noticed observe your BP and to do some blood tests (FBE,
that he had typical symptoms again, so she sent him to the GP Risperidone blood level). I will call in the psychiatric
who changed his medications to risperidone. team to review your condition. They might adjust the
dose of your current medication or they might decide
Task to switch to another one. If they decide to change,
a. History (haloperidol x 10 years risperidone; can then you will need to stay in the hospital for the
talk to his dead brother recently; dizziness on standing crossover period which takes around 2 weeks.
and sitting; - Meanwhile, I would like you to be aware of general
b. Physical examination (oriented to person, place and rules to be followed when taking an antipsychotic
time; no pallor, dehydration, jaundice, BP sitting medications:
120/80 100/60, BMI N) o Take medication exactly as prescribed
c. Diagnosis and Management o Have a routine to take at the same time
everyday
History o If you forget to take a dose, take it within the
- Is my patient hemodynamically stable? next few hours. Otherwise, skip the dose.
Please do not double the dose.
29

o You need to continue taking the medications o When ECT has been successful in the past
even if you feel well o Last resort to treatment-resistant OCD,
o Never stop the drug abruptly Parkinson disease, tourette syndrome
o Inform your doctor if you develop any side - Contraindications: raised ICP is the absolute
effects, other symptoms, and before taking contraindications.
any other medications - Some specialists believe that it is more effective than
o Half-minute rule (30sec): when you get up in drug therapy. It is amongst the least risky medical
the morning, sit up slowly and stay for 30 procedure carried under GA.
seconds, put the legs over the side of the - Extra caution is required in a number of clinical
bed for 30 seconds and slowly get up and situations according to the medical history of your
walk prevent any dizziness in the future mother (recent MI, cardiac arrhythmia, pace makers,
hypertension, intracranial pathology, epilepsy,
Counseling on ECT osteoporosis, aneurysm, skull defect, retinal
detachment
Case: A 55-year-old lady has been admitted to the hospital for - Pre-ECT evaluation:
severe depression with suicidal ideation. Her son has come to o Full medical history, physical examination,
your GP clinic to discuss about ECT as the psychiatrist has routine investigations including fundoscopy
recommended it as the best choice of therapy for his mom at will be done to make sure ECT is safe for
this stage. your mother.
o Anesthetic consultation for risk of
Task anesthesia
a. Discuss about ECT o Explain of procedure and informed consent
b. Answer questions o Patient should be fasted for 6-8 hours
before the procedure
- Painless - The procedure is carried out under the supervision of
- Under GA and with muscle relaxants a consultant psychiatrist and anesthetist. Patient is
- Indications given general anesthesia and muscle relaxant to keep
- Contraindication: her relaxed and unaware of seizures. Small devices
o Increased ICP called electrodes will be placed on specific locations of
- Side effects: her head to give a series of brief electrical pulses.
o Anesthesia SE Patients vital signs are continuously monitored and it
o Muscle relaxant SE takes about 10-15 minutes for the procedure to
o Memory loss and transient confusion complete and patient is taken to recovery area for
- Drug Interactions post-anesthetic care.
o Non benzodiazepines for anxiety - Duration of treatment: typically3x/week for 6-9
zolpidem, zolpidone, zolfresh treatments depending on the patients condition and
o No antidepressants and antiepileptics response to treatment. It can be performed as an OP
o Mood stabilizers may result to postictal procedure or when the patient is hospitalized. The
confusion but does not interfere with efficacy symptoms start improving after 2-3 treatments.
- Consent: - Side effects:
o Normal MMSE o Immediate: Headache, muscle pain, nausea
o Not under effect of drugs/alcohol and drowsiness are benign and should
o Age respond to symptomatic treatment
o Post-ECT delirium needs close supervision
- Is the son permitted to discuss the condition of his and supportive treatment and IV
mother? I understand that you are here to discuss psychotropics if required.
about ECT: its indications, contraindications, o Memory impairment that usually resolves by
procedure, side-effects and post ECT management 4-6 weeks following treatment
- Any particular concern? Please dont hesitate to stop - Maintenance treatment: ongoing treatment will be
me if you have questions. required to prevent a recurrence in the form of ECT,
- It is a medical procedure in which a series of low- antidepressants, psychotropics, CBT or
frequency electrical pulses are passed through the psychotherapy.
brain to produce brief-controlled fits. It can provide
rapid and significant improvements in severe CHILD AND ADOLESCENT BEHAVIORAL DISORDERS
symptoms of a number of mental health conditions
and doesnt cause any structural brain damage. Autism
- Indications
o Psychogenic depression Case 1: You are GP and your next patient is a 3-1/2 year old girl
o Melancholic depression unresponsive to with her mother who comes to you because she is concerned
meds about her destructive behavior. She has been contacted by
o Psychosomatic depression (Life-threating childcare who says she is different from other kids. A hearing
refusal to food or severe psychomotor test has already been done.
disturbance)
o Severe postpartum depression and Task
psychosis a. History play for hours, specific game, likes TV
o Catatonic schizophrenia cartoon, energetic; doesnt want to sleep; looks fine;
o Severe mania talked late; lack interaction to other kids; not hearing
o Pregnancy when medications cannot be mom; lacks social input
given as they can cause harm to fetus b. Explain diagnosis
o Elderly people who cannot tolerate drug side c. Counsel accordingly
effects
30

Case 2: You are a GP, a 5-years-old girl was brought to you by please understand that there is a lot we can do to help
mom because childcare complained that she has destructive her. The actual cause of autism is not known, but
behavior and claims she is different from other kids. She has a there is a genetic tendency. It is associated later on in
6-year-old brother with whom she does not interact well. Her life with epilepsy, OCD, sometimes intellectual
speech was delayed by 1 year. Her hearing and vision tests are disability and Tourette syndrome. This condition is
normal. She has been to the specialist who diagnosed her with more common in males. Around 1:1000 australians
autism. The mom is not clear about the diagnosis. suffer from autistic spectrum disorder (autistic
disorder, high-functioning autism, asperger syndrome,
Task pervasive developmental disorder). Autism is very
a. Explain the diagnosis to the mom difficult to diagnose under the age of 3. If any
b. Counsel accordingly suspicion is raised, it is important to establish the
diagnosis to help with treatment. The earlier the
History treatment is started, the better is the outcome. I want
- Please explain what you mean when you say that you to be aware that your daughter might exhibit
shes different from other kids. Is she aggressive? temper tantrums or obsessions. She will be resistant
How is her interaction with the other kids and with the to a change of daily rituals. She might be sensitive to
childcare workers? Does she exhibit any repetitive some colors, textures or smell. Otherwise, physically,
behavior? Does she become upset when her routine she will be a healthy and well-developed child. We do
is changed? Do you think she is particularly attached not expect for her to have the same emotions and
to a certain toy/object? Does she prefer to play alone? moods as everyone else. In a multidisciplinary team
Is she able to initiate play with other kids? How does approach, we will start an interventional program
she respond when you or someone else call her? Any which includes a. behavior modification therapy, b.
repetitive use of a word or sentence? Is she able to speech therapy, c. education, sensory and motor
maintain eye contact? How many friends does she program, d. regular medical checkups e. regular
have? Does she respond appropriately to changes in hearing and vision checks. You and your family need
your mood? to be involved throughout the program. Most of her
- BINDS: Please tell me more about your pregnancy? behavioral problems can be improved with this
Any complications? Mode of delivery? Any problems program and 5% of these kids go on to live an
like breathing? Did she require resuscitation? Can you independent life. However, the majority will require
tell me when she started to speak? When did she start life-long support.
babbling (6-8 mos)? Proper words (10-12mos)? Can - No special school except for asperger (do not want to
you tell me how much she can speak now stigmatize patient)
(words/sentences)? Developmental milestones: walk,
eat, drink independently? Does she smile socially? Behavioral Problem (ADHD)
Immunization? Any other medical or surgical
conditions? Case: You are a GP and a 6-year-old boy was brought to you
- Any family history of mental retardation? Autism? Or by his mom because she says the child is very active and the
other developmental disorders? teacher has complained that the child is loud and disruptive in
class. He is in grade 2.
Differential Diagnosis:
- ADHD Task
a. Focused history
Diagnosis b. Diagnosis and management
- Most likely, your child has some kind of a behavioral
disorder. The most common one to present this way is Differential Diagnosis for Hyperactivity
called autistic spectrum disorder. - ADHD
- It is characterized by: - Asperger Syndrome
o impaired social interaction - Oppositional defiant disorder
o speech and language problems - Visual and hearing problems
o abnormal repetitive behavior - Trauma (head injury)
- It is important to confirm this diagnosis preferably by a - Developmental problems
specialist. Unfortunately, it is a lifelong condition that - Previous infections (meningitis,
cannot be cured, but it can be very well controlled with - Home/school problems
appropriate interventions. I will refer you to the speech - Physical or congenital lesions
pathologist once the diagnosis is confirmed. Autism is
associated with a high risk of epilepsy and OCD so History
there is a possibility she might develop fits or seizures - Can you tell me more about the problem? What do
later on. There is no medical treatment but regular you mean when you say that he is disruptive and loud
checkups are important because these kids never at school? Is he aggressive towards his classmate? Is
complain. You will have all the support from me as it difficult for him to wait in lines? Is he pushing around
your GP, pediatrician, child psychologist, speech other kids? How is his academic performance?
pathologist, centrelink, and Autism Association of - Please tell me more about his behavior at home. Do
Australia. Please come back once the diagnosis is you think that he is able to concentrate on a given task
confirmed and we can talk about it in more detail. for at least a few minutes? Does he pay attention to
what goes around? Does he pay attention to
- Case 2: I can see from the notes that your child has commands? Does he talk rapidly without finishing
been diagnosed with a developmental disorder called sentences? Do you think he is able to finish a task
autism. It is a lifelong condition where the childs skills given to him? How long can he play with a particular
of social interaction and communication are affected. new toy? Do you think hes impulsive? How many
She will show repetitive behavior. She might have hours does he sleep at night?
speech problems. I understand that it might be
shocking and distressing for you to hear all this, but
31

- How is his relationship with you or with his family? - These kids might later on develop conduct disorder.
How is your home situation? Any recent changes at The usual age is preschool.
home? Who takes care of him most of the time? Does - Criteria
he go to child care? How is your relationship with your o Persistence of stubbornness
partner and other kids? Any new relationship for you? o Refusing to comply with instructions
How many kids do you have apart from this one? o Unwilling to compromise
Hows the behavior of the other kids? How is the o Deliberately testing the limits
relationship between them? Any family history of o Failure to accept responsibility
similar problems? Do you think he was diagnosed with o Blaming others for their own wrongdoings
a medical condition? How is he doing now? o Deliberately annoys mothers
- How was your pregnancy with this child? Any o Frequently losing temper
problems with delivery? Any history of head trauma or - Management:
brain infections or any other illnesses? Has he ever o Improved parenting skills (conflict resolution,
had a formal hearing or visual test done? How is his communication and problem solving with the
immunization? Are you happy with his growth and child)
development at this time? Are you happy with his diet o Anger management skills for the child
or nutrition? o Family counseling
o Classroom strategies (social skills
Management development sessions)
- Most likely, because of your concern and because of - Appear well in front of other people such as doctors
the teachers complaint, I do suspect that your childs except people they know well and whom they consider
behavior is different from others. I still need to have as authorities
his vision and hearing checked by a specialist. Before
labeling a child as having a behavioral problem, it is Differential diagnosis
important to obtain assessment reports from school as - Oppositional defiant disorder - argumentative,
well as from the family. This is called psychosomatic stubborn, picks up a fight
testing best done by the specialist child psychologist. - ADHD
- The most common behavioral problem in this age - Conduct disorder - extreme; gets into physical fights,
group is called ADHD (attention deficit hyperactivity theft/fire, cruel to animals and people, no remorse; no
disorder). It is a developmental disorder that results in relationship
poor concentration and lack of impulse control. Please
understand it is not a physical illness, but it can affect Features:
the childs learning and social skills. Usually, there are - Easily lose temper
associated problems with family function. - Refuse to follow rules
- Once the diagnosis is confirmed, we will start - Deliberately annoy others
treatment that includes behavioral modification that is - Blame others for their own mistakes
done by the psychologist. There are classroom - Can be verbally hostile but not physically
strategies to help with his learning and concentration - Acts are usually directed to those who are well known
span. There are special teachers who are qualified to to them
run these programs. Family counseling is also - Parents are facing numerous arguments
required. The fourth aspect is medications. These
medications are usually prescribed by the pediatrician VS Conduct disorder
only and are basically stimulants (methylphenidate - Aggressive
ritalin, dexamphetamine or atomexetine). The - Involve in bullying, fighting, theft, fire-setting,
single most effective treatment for ADHD is - Do not follow rules
methylphenidate. They stimulate areas of the brain for - Property destruction
impulse and concentration. Side effects are reduced - Antisocial
appetite and growth problems. - Can be cruel to animals
- Don't have any remorse
Oppositional Defiant Disorder - Refer to adolescent mental health service

Case: A mother brought his 8-year-old son with problems in Management: Behavior modification and family assistance
behavior.
History
Task - Can you tell me a bit more about it? (I'm concerned
a. History for 8 minutes (problem with School about his behavior. He is argumentative and stubborn
performance, argumentative, disobedient, picks up and is getting more difficult for me to handle him). For
fight with other kids and has problems getting along how long has he been having this problem? it's been
with them, doesn't listen to teacher's commands and an ongoing problem since he's very young but
does what he wants to do; has a sister who is fine; increasing with age). Does he lose temper very
fights with sister; no bullying in school noted; full-term quickly? Yes he gets annoyed with his siblings while
planned pregnancy; no complications; immunization playing with them. Does he listen to what you say to
up to date; separated from husband 4 years ago but him? No. He is disobedient and when I ask him to do
dad visits regularly and has good relationship with something he gets angry and does not do his work.
dad) Does he have any other siblings? Any problem with
b. Most likely diagnosis them? No. He has 2 other siblings without problems.
Des he go to school? Did you talk to his teacher about
DSM IV Criteria it? Teacher has similar complaints. He doesn't listen
- Repetitive persistent pattern of opposition, to teachers and difficult to settle him down. Does he
disobedience, and disruptive behavior towards have friends in school? He has but he doesn't
authority figures persisting for at least 6 months. have good relations with them. How is his
32

performance at school? His p is affected though he Diagnosis and Management


enjoys going to school. Is he a hyperactive and - You have made a very good decision to come here
restless child? No. Is he able to concentrate on what and discuss about your son. It is important to find out
he is doing? Yes. Is he Impulsive and often acts the cause that is affecting your childs performance.
without thinking? No. Has he ever been very - Confidentiality statement. I am here to help you.
aggressive and destructive in behavior? No. How is all - The reason why your sons school performance has
this affecting your life? I am very tired and concerned deteriorated could be due to stress at home, school
about this situation and need your help. bullying, organic reasons, hearing or vision problems
- How is his general health? Any problem with his vision or developmental problems. In your case, most likely it
and hearing? Any learning difficulties or mental is because of the family situation. I would like to
slowness? Any other developmental problems or organize a family meeting. If there are any marriage
disabilities? problems, I can refer you to the marriage counselor. If
How is the situation at home? Any financial issues? there are any financial problems I can arrange with
Are you able to take care of your children? Yes. centerlink or if you need any other support, I can
BINDS? Did you have any problems with his delivery? organize a social worker.
Did you notice active movements when he was in your
womb? is his immunization up to date? Any previous - I would like to arrange a meeting with the child if it is
hospitalization for a serious condition? Medication? possible.
Allergies? FHx of behavioral problems? Do you have - I would refer you to the specialist pediatrician for
any idea of his father's childhood behavior. formal hearing and vision assessment.

Diagnosis and management Slow School Performance/Homosexuality/Adolescent


- From history he has a condition called ODB. I need to Counseling
assess the child's behavior and will arrange referral
for full assessment by a pediatrician and psychologist Case: You are a GP and a mother brings her 15-year-old son
to confirm the diagnosis and rule out other behavioral claiming that her sons school performance from bad to worse
disorders. for the past 6 months.
- Advice: Notice and reward the acceptable behavior by
praising him. Ignore minor irritating behaviors. You Task
can withdraw privileges like watching TV or playing a. Counsel the mom
video games after the behavior occurs for about 1-2
days without compromise. Smacking should be - I understand you have come to see me because you
discouraged. are concerned about Johnnys school performance.
How is everything at home? Any recent financial or
Slow School Progress emotional problem? Is Johnny your only child? How is
the relationship between siblings? Has he changed
Case: You are a GP and an 8-year-old boy was brought by school recently? When did you talk with his teacher
mother because of slow progress at school recently. last time?
- If you dont mind, I would like to talk to Johnny
Task separately. He is nearly an adult and it will be a good
a. History opportunity for him to learn how to explain his
b. Manage the case problem.
- First of all Johnny, our discussion is private and
Differential Diagnosis confidential unless yours or other lives are at risk. In
- School Bullying case I need to report to the authority I will only divulge
- ADHD relevant information but not all consultation details.
- Absence seizure - Johnny why do you think your school performance
- Hearing and Vision problems getting worse? Have you been subjected to any
- Child abuse bullying? Is there something on your mind that worries
- Psychosocial factors you (I like another boy in my class)? Do you feel
sexually attracted to him (Yes)? Does he know about
History it (No)? How does this situation make you feel (I am
- When did it start? Any change in his behavior at lost. Am I gay, etc)? Johnny, lots of young people
home? Is he very noisy or disruptive? Does he disturb like people of the same sex. Studies have shown that
you quite a lot while youre talking on the phone or between 8 and 11% of young people in Australia
doing something else? Is it very hard to keep him experience same sex attraction. Sexual and romantic
attentive? Does he sit and watch movies or play relationships between people of same sex have been
games at home? Did you talk to the teacher about found in every known culture and society. You are
similar behavior in school? Any rigid routines or going through a phase where youre trying to work out
repetitive behavior? Any funny faces or hand your social identity. It doesnt necessarily mean that
movement or jerking? Are there times when you you are gay because people often experiment with
realize that he is not responding to you? Is he happy their sexuality; but remember, being gay is a natural
to go to school? Does he have a lot of friends? Did he and normal sexual identity. There is nothing wrong or
ever talk about being bullied in school? Any concerns abnormal about it.
about hearing and vision? Did you get his hearing and - I just need to ask you a few questions. Hows your
vision checked? Any concerns about his growth and mood? Sleep? Appetite? Do you enjoy things which
development? Immunization? Any problems at home? you used to like? Do you socialize? Do you have
Financial? Are you a happy family? How is his hobbies/interest? Do you have a good friend? Did you
relationship with you or his father? Are there any talk to anyone else about it? SADMA? Are you close
stresses in the family? SADMA (parents) to your parents? Have you ever discussed sexuality
problems with them? Are you going to tell my mom?
33

No, but remember, your parents love you and worry Risk Assessment
about you. Eventually, you should tell them or at least - Static Factors
explain that you are going through a difficult stage. o Age (elderly or very young age groups)
They are here to support you. o Past history of suicide
- I want to refer you to Family Planning Victoria. They o Past history of any medical illness including
are specialized in sexuality and reproductive health. depression and psychosis
There is a lot of help and support for you. o Child abuse
- I would also like to see you in a weeks time. - Changing Factors/Circumstances
- Doctor whats wrong with Johnny? Johnny is going o Patient living alone
through a difficult stage. There are physical and o Stressor (uni, work, family)
mental changes in adolescence which often create o Relationship
inner conflict which we will try to help Johnny to solve. o Access to means
I will see him in a week time. o Drugs/intoxication disinhibited and leads
to patient being impulsive
RISK ASSESSMENT
- Ask about the INTENTION while patient was cutting
Suicidal Attempt herself. Was it to commit suicide or to get physical
pain?
Case: Your next patient in ED is a 45-year-old man who was - When to admit
involved in a high speed single car crash earlier today. Other o Suicidal ideation
drivers reported that he was speeding along the highway and hit o Psychiatric diagnosis (depression or
a tree. Air ambulance was deployed and he sustained minor psychotic)
injuries with Colles fracture. He was brought by ambulance and
was assessed by the trauma team. His injuries have been taken Management
cared of and they have put a plaster for the fracture. Now the - Ensure confidentiality
resident is handing this patient over to you for final checkup - Appreciate that the patient is in stress
before discharge. - I understand that you are going through a tough
phase, can you please talk more about it. I know I am
Task sorry to hear that. Sometimes, we are very frustrated
a. History (intended to kill herself; divorced 6 months and some people do make us sad, but did you have
back and lost her job; worked as an accountant any intention to kill yourself by cutting your thigh (no
before; had repeated attempts; overdosed before; doctor, it just makes me feel better)? Have you done
generally a sad person) this before? Have you ever thought of harming your
b. Talk to patient and management boyfriend or any body else?
- How is your mood? Do you feel sad? Do you think you
Highlights: have lost interest in things that you used to enjoy
- CONFIDENTIALITY before? Any sleep problems? Change in appetite or
- What was the patients INTENTION? Did you intend to weight? Do you think life is worth living? Have you had
kill yourself or was it to do self-harm? problems with your relationship in the past as well?
- Have you had any previous attempts at self-harm? Are there any times when your mood is really high?
- Have you ever been diagnosed with any mental - Ill ask some questions which might seem funny but do
illness? you feel/see/hear things that others do not? Do you
- Have you seen any psychiatric services before? have any strange experiences?
- Did you regret doing it? (If intention was suicidal) - Do you think Im crazy? Sorry, I didnt mean that, but
- Past, present and future! these are routine questions for all patients who are in
your situation.
Risk Assessment (Borderline Personality Disorder) - Whom do you live with? Do you get along with them?
No issues? How about your parents? Do you have
Case: You are an HMO in the ED and your next patient is a24- friends? Do you socialize with them? Do you work or
year-old lady who has a history of repeated self-harm. She has are you a student? Any problems at work or at the
been diagnosed with borderline personality disorder. She cut uni? Is there anything else that is bothering you
her thigh this time. She was found intoxicated and was brought (financial, relationship, etc)?
to the hospital. Now she is okay. The wound has been taken - SADMA?
cared of and she wants to go home. - Ask about insight, cognition and judgment.
Task o Do you think there is something wrong? Do
a. Risk assessment you think you need professional help?
b. See if shes ready to go home o Fire/envelope question
o Do you know where you are, date, and time.
Case 2: A 16-year-old girl was at a party and had - PMHx: thyroid diseases?
benzodiazepine overdose yesterday after having a fight with her - Just let me go home! I know that you are really
boyfriend. She lost consciousness and was brought to you by stressed. I am here to help you, so please just bear
her friends. Now, she is ready to be discharged and your task is with me and help me so I can help you.
to do the risk assessment. - You can go home Mary. How would you go home?
(on history she takes tablets from mom to be able to sleep in When you go home, what will you do? What are your
this case advise on sleep hygiene!!!!). plans tomorrow? I would recommend you to call your
family or friend to come and pick you up. I would not
Case 3: You are an HMO in the ED and a 22-year-old female recommend that you drive now. I am really concerned
has been admitted with a number of wounds in her arms and with the way you are coping up with stress, so I will
legs. Most of them were superficial cuts. She has been treated refer you to the psychiatrist and psychologist. They
by the registrar. The wounds have been sutured and she wants will do talk therapy and will teach you techniques on
to go home. how to handle stress without harming yourself.
- About alcohol we need to discuss about the safe
level of drinking.
34

- If you agree to it, I would be happy to arrange a talk refer you to a counselor or psychologist with whom
with your friends or family. There is also a 24/7 hotline you can talk about things and teach you how to handle
number 1800187263 (1-800-18-SANE). Anytime you stresses of your life. I would also like to refer you to a
feel stressed, you can give them a call and you can psychiatrist for formal assessment.
talk to them. - At this stage, you can go home but I would like to give
- If you have financial problems centerlink you advice on sleep hygiene:
- Organize social worker if you need support o Have a regular pattern of sleep
o Dont have coffee, tea, or heavy meals
Benzodiazepine Overdose o Hot shower or milk
o Dont watch TV before going to bed
Case: You are an HMO in ED and a 16-year-old girl comes who o Bed should be used for 2 purposes: sleep
overdosed with mothers benzodiazepine tablets. Her mother and sex
found her and brought her to you. She has been resuscitated - If you are stressed or upset, then talking with a
and now is stable. Her mother is waiting outside. counselor might help
- Regarding your alcohol, please book an appointment
Task with your GP so he can advise you on the safe level of
a. Talk to the patient (started when parents got divorced; drinking
had difficulty sleeping for a few months, did not have - Social support.
intention to kill herself; school performance or grade;
mood is okay, I dont need help, why would I need ABUSE AND VIOLENCE
help. Doesnt think the TV is talking about her;
b. Decide on further management Child (Sexual) abuse

Risk Assessment You are a GP and a 4-year-old girl was brought in by mom who
- Dynamic: says she has a rash in the genital area. Her mom is divorced
o Patient: mental state, diagnosed mental and mom lives with a new boyfriend for the last 5 months. The
illness (depression, psychosis, mania), girl usually spends weekends with her dad. This weekend, she
intention/remorse, has refused to go to dad.
o Context/circumstance: access to
means/weapons, accommodation, family Task
support and friends, stressors (financial, a. Counsel the mom
personal)
- Static: History
o Time: history (have you done it before), Hx - I have been told you are worried that your daughter
of mental illness, developmetal history (child has a rash in the genital area. I need to ask you a few
abuse, conduct disorder, personality more questions if its alright with you.
disorder) - Mention confidentiality
o Place: culture - Can you please tell me more about what happened?
When did you notice the rash? Is it getting worse? Did
Counseling your child tell you what happened? Why do you think
- I understand you took some of your moms tablets, she refuses to go to her dad? Have you noticed any
how are you feeling now? Are you feeling better? behavior that is different than usual in your child (e.g.
- I am sorry to hear that. nightmares, irritability, refusing to eat or drink, refusing
- Confidentiality to talk to you)? Have you noticed her in an abnormal
- Was it accidental or did you take it with intention? Did position (like knee chest position)? Do you think she is
you have any intention to harm or kill yourself? How is enjoying the usual activities or is she withdrawn?
your mood? Do you still find things pleasurable? Do - Waterworks? Does she cry when she passes urine?
you go out with your friends? How are things with Bowel habits? Allergies? Itching down below?
school or uni? How is your school performance? I Bleeding? Discharge? Any possibility of trauma or
understand you have sleep problems. Is it difficulty foreign body? Is this the first time to have this rash?
getting to sleep or waking up in the morning? Hows - I need to ask you some questions regarding your
your appetite? Any changes in your weight? Do you home situation that will help me to better understand
think? What do you think about life? Do you think it is your childs condition. How is your relationship with
worth living? Have you ever thought of harming or your ex-husband and with your new partner? Any
killing yourself? Are there any times when your mood stress? Any violence at home? How often does she
is very high? Whom do you live with at home? Do you spend time with your boyfriend? Did you call your ex-
have enough family support? Are you a happy family? husband to ask him what happened? Do you know
Have you tried reaching out to your mom? If youre about his family situation now? Are you aware if he
happy I can arrange for a family meeting. Any other smokes, drinks, or uses recreational drugs? Do you or
stressors like financial or school/work? your partner smoke, drink or recreational drugs?
- How is your general health? Do you have any weather
preference? Differential diagnosis
- Do you hear/see things that others do not? Do you - Child abuse
have any strange experiences? SADMA? - Foreign body
- Allergy
Management - Trauma
- At this stage, Im glad you dont have intentions to - Vulvovaginitis
harm yourself, but you are upset because of your
parents divorce. If you like, I am happy to arrange a
family meeting and talk about things. I would like to
35

Management unplanned pregnancy. The father left before she was born. Now,
- At the moment, I do have a few differentials in mind they live with her new boyfriend who is unemployed. The mom
like vulvovaginitis which is an infection of the genital works part time and the boyfriend looks after the child. You
area, or skin allergies, but because you are worried examined the child and found a bruise on her left cheek, a
about possible abuse, it makes me concerned as well. painful swelling of her right upper arm. An xray was done that
I will notify the child protection authority as well as shows a spiral fracture of the right humerus.
VFPMS (Victorian Forensic Pediatric Medical
Services) or pediatric department of the local hospital. Task
- It is better for them to come and talk to your child as a. Explain diagnosis to mom
they have a special way of doing it. If required, they b. Discuss appropriate management
will do a genital examination preferable by an
experienced pediatrician. They might take necessary Problem list:
swabs and samples. Please dont talk to your child a. Identify child abuse. Exclude organic causes.
about any of this (mental trauma/might influence the b. Assure confidentiality
child). It is a long process to find out what happened. c. Assessing the psychosocial status
You and your daughter will have all the help and d. Delicate, empathetic explanation of the situation.
support throughout the way. Convince her for hospital admission
- They might admit the child to the hospital for
investigations as well as possible treatment of the Differential diagnosis:
rash. She will be seen by a child psychologist as well - ITP
as the social worker. We will involve the police if - Pancytopenia
required, but the Child Protection Authorities will be - Leukemia
the first one to be notified. - Bleeding disorders

Guidelines (VFPMS AKA Gatehouse Centre): History


- All suspected cases of child sexual abuse must be - Please tell me more about the bruise? When did you
referred in the first instance. notice it? Is it the first time you noticed the bruise?
- The genital examination must be limited to Any other marks anywhere in the body? How do you
INSPECTION only to check the amount of bleeding, think this happened? Do you think the baby has been
the extent of the rash or any discharge only with the crying since then? Does he have any fever, cough, or
cooperation of the child. Complete examination will noisy bruising? Do you think hes feeding well? Is he
only be done by a trained consultant. GPs task is to: breastfed or bottlefed? Any bleeding from anywhere in
a. control bleeding, b. collect any evidence on the the body? Any change in his bowel habits? Any
child including clothing, hairs, c. make the child as change in the number of wet nappies? Has he been
comfortable as possible, d. provide emotional support putting on weight since birth? Is there a family history
for the family, e. photographs if required of bleeding problems? Is this your first child? Any
- Call anytime: 24 hours a day pregnancies or miscarriages before? Was this a
planned pregnancy? How was the delivery? Normal or
Child Abuse cesarean? Was it a full term or premature delivery?
Any problems or complications after birth? Are you
Case: You are the HMO in the ED. Your next patient is a 24- satisfied with his growth and development? At the
year-old mother with her 4 month old baby boy John. She moment, is your child able to sit with support? Does
complains that her child is crying non-stop. You notice some he smile when he sees you? How is his
bruise marks on the babys body which she claims occurred immunization? May I ask how, is the home situation?
when Johnny rolled off the change table while getting his nappy Are you in a stable relationship? Any conflicts at
changed while under her boyfriends care. home? Do you work? Does your partner work? Any
financial problems? When you go for work, who takes
Task: care of the child? Does he go to childcare? Do you or
a. Further detailed history your partner smoke, drink alcohol, or use any
recreational drugs? Do you have any support from
your family, parents or friends?
b. Examination findings
c. Further management Physical Examination
- General appearance: conscious, active, alert, or
Case 2: A 23-year-old female presents to the ED with her 6 irritable, crying, drowsy?
months old baby boy. The child has a bruise on the left cheek - Growth chart for height, weight, and head
and has been crying for a very long time. The mom thinks he circumference
may be suffering from a bleeding disorder. She is worried. - Vital signs
- Skin of the whole body for any bruises, fingermarks,
Task areas of burns or scalds, cigarette burns
a. Further history (child fell from changing table, - Anterior and posterior fontanelle for any bulging
unemployed, unplanned pregnancy, recently worked (shaken baby syndrome rupture of cranial arteries)
part-time, smoker, lost all family support financial - Eyes: conjunctival hemorrhages
crisis) - Mouth: rupture of frenulum (forceful feeding)
b. Physical examination findings (femoral fracture) - Lymph nodes
c. Management - Chest and heart
- Palpate all ribs for possible fracture
Case 3: A 19-year-old mother brings her 6 months old daughter - Palpate all bones of upper and lower for swelling,
to your GP. She is complaining that her daughter cries a lot. She tenderness, deformity
was born at 26 weeks of gestation. Her BW was 970 grams.
She stayed in the hospital for about 3 months. It was an
36

Investigations - Confidentiality. I understand that its your personal


- FBE, coagulation profile, complete skeletal xray or matter but let me reassure you that
bone scan (provide information regarding new and old - Are you in a stable relationship? Do you have a
fractures); CT scan of the head and USD of abdomen partner? How is your relationship with your partner?
and pelvis Sometimes, partners react strongly in arguments or
may use physical force. Is this happening to you by
Management any chance? Have you ever been scared about your
- After doing thorough examination on your child, it partner? Dont worry were here to help you. You need
seems like the nature of his injuries does not correlate not be afraid.
with the mechanism of injury. We call this non- - How often does this happen? When did it all start? do
accidental injury. It is better if we admit your child to you have enough support from family or friends? Do
the hospital where he will be seen by a specialist you have other injuries apart from the one in your
pediatrician. We need to do some tests to rule out a head? How is your mood? Do you still find things
bleeding disorder and any other injuries to the rest of pleasurable? Sleep? Appetite? Weight? Do you think
his body. I want you to understand that this might be a life is still worth living? What is your perspective
grave situation for your child. He needs urgent towards life and towards what is happening to you?
medical treatment. Have you ever thought of harming or killing yourself?
- I understand that it might be hard for you to take care SADMA? Do you have any strange experiences?
of this child especially because you are young, - Hows your general health?
working and studying. Our job is to help you out as
much as possible by providing you with appropriate Physical Examination
referrals and resources. I need to inform the Child - General appearance
Protection Authorities (or VFPMS), but I want your - Vital signs
cooperation to ensure your childs safety and - Are there any other scars or bruises. With the patients
wellbeing. We will not take away the child from you. consent I would like to take some pictures for record
The CPS will interview you and your partner and purposes
provide the necessary support system for you and - Chest, heart, and CNS
your child.
Management
Dr. Wenzel - From the history, I am really sorry to hear whats
- ADMIT happening between you and your husband but I really
- Report to Child protection authority praise your efforts. You are a very brave woman. This
- Maintain empathetic approach to parent is known as domestic violence and it is not
- Explain further investigation will be conduct acceptable. In my opinion, I believe you are not safe
- Explain support with the situation and in your house. There are a lot of
- Possible photography for evidence organizations and support groups to get a safe place.
They will help you financially as well and offer you
Domestic Violence shelter and a place to live.
- I will also organize a social worker. I would like you to
Case: Your next patient in GP practice is a 24-year-old Fiona. see a counselor to talk about what happened.
She has visited you 4x for the last 6 months. The first she came - I would recommend for you to stay in a separate room
to you with 5% burns on left hands. The second time she had and dont hesitate to call police if at any time you feel
some injury on the right hand and the other 2 consultations, she its getting very serious.
complained of tiredness for which she was thoroughly - I would also like to see and examine your babies as
investigated and everything was normal. Today, she is visiting contact the Child Protection Services to assess your
you with a complaint of injury on her forehead. situation. I would give you the numbers of the
domestic violence support groups.
Task - If you think your partner agrees, I am happy to
a. History (I was running after the kid and hit my head arrange a family meeting. If you want, I can contact
on the door, he is very short-tempered and has lost the social workers or police on your behalf. But I
his job because of it; and when he comes home he would like to recommend you to do that because in my
starts throwing things and pan hit me especially when opinion you are not safe in your own home.
he drinks; at least 2x a week, beating started after - Before you leave, I would like to dress your wound
birth of the 3rd child when they had experienced and give you painkillers.
financial problems; no support; have trouble with
sleep) LEGAL AND ETHICAL ISSUES
b. Physical examination(distressed lady; 2x2 cm
laceration on the forehead with erythema and Confidentiality
swelling; tender)
c. Diagnosis and management Case: Mr. Smith comes to your GP clinic because he wants
some advice regarding his wife Joana (72-years-old who's
HELP Mnemonic becoming very forgetful for the last few weeks. Mr. Smith is
- Hear establish pattern and onset; how bad concerned because last week Joana went to the shopping
violence is center and forgot to turn off the oven, and before that she lost
- Esteem the house keys. You have seen Joana 4 days ago for simple
- Life situation cold. Mr. Smith wants to know if you can recommend a nursing
- Praise home placement for her.

History Task
- I understand that youre getting injured quite often and a. Counsel accordingly
now you have an injury on your forehead. How did it
happen?
37

Counseling - We continue paracetamol even when we reach to


- Confidentiality stage 2 or 3. Often when we move to strong opioid
- Come again with wife use, we consider to give you slow release medication
- Cannot divulge information about the wife once twice a day to give you a good background
- What about you Mr. Smith? relieve plus top ups with short acting medications or
- I am happy to see her tomorrow about her condition break through pains (endone). Often with opioids we
and if she agrees to have you during the consultation will start antiemetic and laxatives to prevent side
then I would be more than happy to assess her. effects such as nausea, vomiting and constipation:
- Nursing home: done by ACAT (aged care assessment Metachlorpromide or Ondansatron up to 8mg TDS.
team) assessment by GP, physiotherapist, Sometimes adding medications: NSAIDs, steroids,
occupational therapist, geriatrician, social worker anxiolytics, antidepressant can improve the effect of
using pain killer
Euthanasia/ End of life request from a terminally ill patient - Also interventional techniques for pain control.
Palliative care specialist might discuss Neurolytic
Book case 124 page 655 Celiac Plexus Block
Case: You are working in a palliative care hospital. Your patient - My throat is very sore and I have problem swallowing!
is Sally aged 65. Who was diagnosed with pancreatic cancer I will examine you. I suspect its a side effect of
despite active medical interventions Sally is now at the end of medication treatment which usually cause dryness
her disease. Sally would like to stop her treatment and return and soreness of the mouth. You can use mouth wash,
home to die. Sally has intravenous therapy, indwelling catheter spray plus local analgesics (lidocaine).
and nasal gastric tube in situ. Sally asks if you would assist her - Ok doctor I see you want to help me but can I refuse
to bring an end to her life. treatment? Sally a competent adult has the right to
refuse unwanted medical treatment.
Task - Can I refuse IV therapy, indwelling catheter and NG
a. Respond to Sallys questions tube? Yes you can, as long as you are competent and
fully understand the consequences for you decisions.
Features of Palliative Care Act: For example IVC makes it easy to deliver necessary
- Reasonable provision of relieve of pain, suffering and medication and hydration but if you dont want it we
discomfort. can remove and look for alternative route to give you.
- Reasonable provision for food and water. It doesnt You can refuse artificial nutrition and hydration but if
include artificial nutrition and hydration. We will you are in pain you cannot refuse pain relieve as its a
provide food and drink if you are comfortable with it. part of palliative care.
- I want to go home. Most of the time we can organize
Counseling it. Let me know more about your family. I have very
- Hello Sally, how are you today? Im very bad doctor supportive family with a husband and two kids. Thats
we all know Im dying, I have thought about it seriously great! I just want to involve Occupational Therapist
for months. I know you have giving me something for and Social Worker.
me for pain. I want you to give me something I can - I will organise community based palliative care to
use to end my life. provide you with all available services and to make
- Sally I can see you are going through a very difficult you as comfortable as comfortable. Sally you are not
time. If the doctor says :I know how you feel. How do alone our aim is to provide the best quality of life you
you know what Im going through? can have with your illness
- Its extremely hard to stay in the hospital far from
home family and friends. I can see it takes a lot of End-of-life request from a terminally ill patient
courage to bring up this conversation. But Sally I
cannot assist you in ending your life. Why not? You Case (Condition 124): Your next patient in GP practice is a 65-
are asking me about active voluntary euthanasia year-old Sally who has been your patient for the last 10 years.
which is illegal in Australia. As a doctor my duty of Few years ago, she has been diagnosed with pancreatic cancer.
care is to provide treatment that will be overall benefit Despite all management, she is now at the end stage of disease
to the patient. And to avoid giving treatment which and has chosen to withdraw from further treatment and remain
may cause harm or suffering. You are supposed to be at home to die. Sally has IV therapy, indwelling catheter, and
my doctor Im not getting to get better Im scared of NGT. She approached and asked you to assist her to bring an
pain you have to be able to do something for me. end to her life. You are doing a house visit today.

- Sally I definitely will try to help you with the pain and Task
make you as comfortable as possible. How bad is a. Respond to Sallys question
your pain? My abdominal pain is progressively b. Answer examiners questions
worsened, pain med made me nauseated.
- Do you know what youre getting for pain relieve at the - What is it that you want me to do for you? I
moment? No, I dont know. Sally I will find out and I understand that you are in a lot of pain and I can only
will involve pain team, or palliative specialist to adjust imagine what you are going through right now, but
your medication. Effective analgesia is possible in up there is still a lot we can do for you. I am sorry but I
to 90% of cases. In general we use analgesic ladder am legally not allowed by Australian laws to help you
approach start from simple pain killer and move to end of life.
weak opioids to strong opioids - Patient does not have the right to stop basic life
o Simple pain killer: Paracetamol, aspirin, support such as pain management, oxygen, and
NSAIDs. parenteral feeding.
o Weak Opioids: Tramadol, codine. - Are there any other concerns you have? I can
o Strong Opioids: Morphine, Hydromorphine organize a counselor for you. If youre a spiritual
(5times stronger than morphine), Fentanyl, person, I can arrange someone to come and talk to
Oxycodone. you. The motive of palliative care is to make your end-
of-life as comfortable for you as possible.
38

What are the ethical dilemmas and legal consequences of


assisting a patient to bring about her own death?
- Physician assisted suicide and active voluntary
euthanasia is against the law and a criminal offense
- Autonomous wishes of a competent patient
- Relief of suffering (adequate analgesia and possible
use of antidepressants)

What alternatives can you offer her?


- Palliative Care

Example Legal Status


Suicide Self-killing (hanging, Illegal:
drug OD) criminal law
Physician- Provision of means to Illegal:
Assisted Suicide kill themselves criminal law
(involvement of MD)
Passive Refusal of antibiotics or Legal:
Euthanasia I: advance directive consent and
Refusal of refusing resuscitation refusal of
treatment by treatment
competent person
Passive Turning off ventilator or Legal:
Euthanasia II: withholding nutrition consent and
Withdrawal or refusal of
withholding life- treatment
sustaining
treatment from
incompetent
patient
Active Voluntary MD administer lethal Illegal:
Euthanasia drug to cause death at Criminal law
patients request
Active non- MD administer lethal Illegal:
voluntary drug in absence of any Criminal law
euthanasia (person request
incompetent)
Doctrine of double MD administer drug Legal:
effect with aim of relieving Criminal law
suffering knowing that (may vary
SE may be hasten between
death states)

Das könnte Ihnen auch gefallen